Sei sulla pagina 1di 67

https://t.me/pdf4exams «Downloaded From» https://t.

me/testseries4exams

Google it:- PDF4Exams


https://t.me/pdf4exams «Downloaded From» https://t.me/testseries4exams

GS SCORE 11
IAS Prelims - 2019

Polity
(Executive + Legislature + Judiciary)

Time Allowed: 2 Hrs. Maximum Marks: 200

RE
Roll No.:

INSTRUCTIONS
1. IMMEDIATELY AFTER THE COMMENCEMENT OF THE EXAMINATION, YOU SHOULD CHECK THAT THIS
TEST BOOKLET DOES NOT HAVE ANY UNPRINTED OR TORN OR MISSING PAGES OR ITEMS, ETC. IF SO,
GET IT REPLACED BY A COMPLETE TEST BOOKLET.
2.
O
Please note that it is the candidate’s responsibility to encode and fill in the Roll Number carefully and
without any omission or discrepancy at the appropriate places in the OMR Answer Sheet. Any omission/
discrepancy will render the Answer Sheet liable for rejection.
SC
3. You have to enter your Roll Number on the Test Booklet in the Box provided alongside. DO NOT write
anything else on the Test Booklet.
4. While writing name and Roll No. on the top of the OMR Sheet in appropriate boxes use “ONLY BLACK
BALL POINT PEN”.
5. This Test Booklet contains 100 items (questions). Each item comprises four responses (answers). You will
select the response which you want to mark on the Answer Sheet. In case you feel that there is more than
one correct response, mark the response which you consider the best. In any case, choose ONLY ONE
GS

response for each item.


6. You have to mark all your responses ONLY on the separate OMR Answer Sheet provided.
7. All items carry equal marks. (2 marks each)
8. After you have completed filling in all your responses on the Answer Sheet and the examination has
concluded, you should hand over to the Invigilator only the Answer Sheet. You are permitted to take away
with you the Test Booklet.
9. Sheets for rough work are appended in the Test Booklet at the end.
10. Penalty for wrong answers:
THERE WILL BE PENALTY FOR WRONG ANSWERS MARKED BY A CANDIDATE IN THE OBJECTIVE TYPE
QUESTION PAPERS.
(a) There are four alternatives for the answer to every question. For each question for which a wrong
answer has been given by the candidate, one-third of the marks assigned to that question will be
deducted as penalty.
(b) If a candidate gives more than one answer, it will be treated as a wrong answer even if one of the
given answers happens to be correct and there will be same penalty as above to that question.
(c) If a question is left blank, i.e., no answer is given by the candidate, there will be no penalty for that
question.

DO NOT OPEN THIS TEST BOOKLET UNTIL YOU ARE ASKED TO DO SO

GS SCORE Google it:- PDF4Exams PTS2019/B-1/102018/11


1
https://t.me/pdf4exams «Downloaded From» https://t.me/testseries4exams

1. Which among the following are the 2. The expression ‘total membership’ means
judicial powers of the Parliament? the total number of members comprising
the House excluding the vacancies or
1. It makes laws to regulate the elections
to the offices of President and Vice- absentees.
President. 3. The special majority is required in Rajya
2. It can impeach the President for the Sabha for the removal of Vice-president.
violation of the Constitution. Which of the above statements is/are correct?
3. It can remove the Vice-President from his
(a) 1 only
office.
4. It can recommend the removal of judges (b) 1 and 2 only
(including Chief Justice) of the Supreme (c) 2 and 3 only
Court and the High courts.
(d) 1, 2 and 3
5. It can punish its members or outsiders
for the breach of its privileges or its
contempt. 4. Which of the following pairs related
Select the correct answer using the code given to different types of Majority are

RE
below: correctly matched?
(a) 1, 2 and 3 only Motions Type of Majority
1. Removal of the Special Majority.
(b) 1, 2, 4 and 5 only
Vice-President
(c) 2 and 3 only
2. Removal of Speaker Effective Majority.
(d) 2, 3, 4 and 5 only
O 3. Motion of Thanks Simple Majority.
4. Removal of Special Majority.
2. Consider the following statements with
Comptroller and
reference to system of representation
Auditor General
SC
in the Parliament.
Select the correct answer using the code given
1. Constitution has adopted the system of
proportional representation in the case below:
of Lok Sabha but not in the case of Rajya (a) 1 and 2 only
Sabha.
(b) 2, 3 and 4 only
2. There is system of territorial representation
for the election of members to the Lok (c) 3 and 4 only
GS

Sabha. (d) 1, 2, 3 and 4


3. The system of proportional representation
does not secure due representation to
5. Under the Indian Constitutional
minorities.
Scheme how the independence of
Which of the above statements is/are correct?
Comptroller and Auditor General is
(a) 1 and 2only secured?
(b) 2 only 1. He is barred from holding any government
(c) 3 only office after completion of tenure.
(d) 1 and 3 only 2. His salary except that of the staff is
charged on the Public Fund of India.
3. Consider the following statements Which of the above statements is/are correct?
with reference to the Special Majority (a) 1 only
provision of the Parliament.
(b) 2 only
1. Special majority constitute a majority that
is, more than 50% of the total membership (c) Both 1 and 2
of the House and a majority of two-thirds (d) Neither 1 nor 2
of the members present and voting.

2
Google it:- PDF4Exams GS SCORE
https://t.me/pdf4exams «Downloaded From» https://t.me/testseries4exams

6. Consider the following statements 9. Consider the following statements


with reference to the Duration of about Public Interest Litigation (PIL):
Rajya Sabha: 1. Under PIL, litigations are taken from
1. The Rajya Sabha is a continuing chamber. those parties that are affected directly
or indirectly however, not on suo-moto
2. The Constitution has fixed the term of basis.
office of Members of the Rajya Sabha for
six years. 2. Landlord–tenant matters can be settled
through PIL.
3. In the first batch of Rajya Sabha, President
was empowered to decide as to who 3. To avoid misuse of the PIL, Court should
should retire. be fully satisfied that substantial public
interest is involved before entertaining
Which of the above statements is/are the petition.
incorrect?
Which of the above statements is/are correct?
(a) 1 only
(a) 1 and 2 Only
(b) 1 and 2 only
(b) 2 and 3 Only
(c) 3 only
(c) 3 Only

RE
(d) 2 and 3 only
(d) 1, 2 and 3

7. Consider the following statements


10. Consider the following statement with
with respect to the Collective
respect to the Money Bill:
Responsibility:
1. It deals with money matters like imposition
1. The ministers are collectively responsible

Lok Sabha in particular.


O
to the Parliament in general and to the

2. Under this if Censure Motion is passed


of taxes, governmental expenditure and
borrowings, etc.
2. The President has authority to give final
decision whether bill is money bill or not.
in Lok Sabha all the ministers have to
SC
resign. 3. It can be introduced in both the houses
but Rajya Sabha cannot amend it.
Which of the above statements is/are
4. A money bill can be a government bill
incorrect?
only.
(a) 1 only
Which of the above statements are correct?
(b) 2 only
(a) 1, 2 and 3 only
GS

(c) Both 1 and 2 (b) 1 and 4 only


(d) Neither 1 nor 2 (c) 2, 3 and 4 only
(d) 1, 2, 3 and 4
8. Which of the following statements
about judiciary is/are correct?
11. Consider the following persons.
1. Every law passed by the Parliament needs
approval of the Supreme Court. 1. Follower of untouchability
2. Judiciary can strike down its own 2. Holder of office of profit under the state
judgment if it goes against the spirit of government.
the Constitution. 3. Having vested interest in government
3. Any citizen can approach the court if his contracts.
her rights are violated. 4. Having age 25 years above.
Select the correct answer using the code given Which of the qualifications given above are
below: not valid for member of legislative assembly?
(a) 1 only (a) 1, 2 and 3 only
(b) 1 and 2 only (b) 2 and 3 only
(c) 2 and 3 only (c) 2, 3 and 4 only
(d) 1, 2 and 3 (d) 1, 2, 3 and 4

GS SCORE Google it:- PDF4Exams


3
Join Telegram groups
https://t.me/pdf4exams «Downloaded From»
https://t.me/testseries4exams

To Boost Your Preparation


PDF4Exams One stop solution for study
Click Here materials of all competitive exams

The Hindu Zone Official


Newspapers & study Click Here
materials

TestSeries4Exam Aall paid test series


Click Here availabble without any cost

All e-Magazines
Estore
in your hand Click Here
Hindi Books
All study materials
Click Here in Hindi

eSandesh (An Indian App)

For More download eSandesh


Google App from play store
it:- PDF4Exams
https://t.me/pdf4exams «Downloaded From» https://t.me/testseries4exams

12. Which of the following pairs of Which of the above statements is/are correct?
parliamentary terminologies are (a) 1 and 2 only
correctly matched?
(b) 2 only
1. Expunction: Deletion of words from the
proceedings or records of Rajya Sabha for (c) 1 and 3 only
being defamatory or indecent by an order (d) 1, 2 and 3
of the Chairman.
2. Adjournment sine die: Termination of the 15. Consider the following statements
sitting of the House which meets again at
the time appointed for the next sitting.
with respect to the Speaker Pro-Tem:
1. The Speaker of last Lok Sabha appoints
3. Casting Vote: The vote casted by the
Chairman in the case of an equality of a member of newly elected Lok Sabha as
votes on a matter. the Speaker Pro Tem.
2. The Speaker Pro Tem has all the powers
Select the correct answer using the code given
below: of the Speaker.
3. He administers oath to the new ministers
(a) 1 and 2 only

RE
only.
(b) 1 and 3 only
Which of the above statements is/are correct?
(c) 2 and 3 only
(a) 1 and 2 only
(d) 1, 2 and 3
(b) 2 only

13. Consider the following statements (c) 2 and 3 only


O
with respect to the oath of Secrecy in (d) 1, 2 and 3
Indian Parliamentary system:
1. The oath of Secrecy should be taken by all 16. Consider the following cases.
SC
the elected MPs 1. Appointment of the Prime Minister when
2. The ministers are constitutionally obliged no single party attains majority after
to take the oath of secrecy after entering elections to the Lok Sabha.
their office.
2. Exercising the pocket veto.
3. The oath of secrecy to the ministers is
3. Sending the bill for the reconsideration of
administered by the President.
Council of Ministers.
GS

Which of the above statements is/are


incorrect? In which of the above cases President is
not bound by the advice of the Council of
(a) 1 only Ministers?
(b) 1 and 2 only (a) 1 only
(c) 3 only (b) 1 and 2 only
(d) None (c) 2 only
(d) 1, 2 and 3
14. Consider the following statements
with respect to the financial powers
and functions of the President: 17. Consider the following statements
related to CAG:
1. Money bills can be introduced in
the Parliament only with his prior 1. President of India prescribes the duties,
recommendations. exercise and power of CAG.
2. He causes to be laid before the Parliament 2. CAG compiles the accounts of the Union.
the Annual Financial Statement. 3. Except Contingency Fund of India, CAG
3. He can make advances out of the audits and reports on all expenditure from
consolidated fund of India to meet any the accounts including Public accounts of
unforeseen expenditure. the Union and of the states.

4
Google it:- PDF4Exams GS SCORE
https://t.me/pdf4exams «Downloaded From» https://t.me/testseries4exams

Which of the above statements is/are Which of the above given statements is/are
incorrect? correct?
(a) 1 and 2 only (a) 1 only
(b) 2 only (b) 2 only
(c) 2 and 3 only (c) Both 1 and 2
(d) 1, 2 and 3 (d) Neither 1 nor 2

18. Which of the following are the 21. Which of the following statements
accountability checks on executive? correctly describes the equality of
1. Vote on Budget by the Lok Sabha. power and status of the Rajya Sabha
to that of the Lok Sabha?
2. Economy Cut and Token Cut.
1. Introduction and passage of all types of
3. Close scrutiny by Departmental Standing financial bills involving expenditure from
Committees. the Consolidated Fund of India.
4. Post Budget auditing by the Comptroller 2. Approval of ordinances issued by the

RE
and Auditor General of India. President.
Select the correct answer using code given 3. Approval of proclamation of all three
below. types of emergencies by the President.
(a) 1, 2 and 3 only 4. A resolution for the discontinuance of the
national emergency.
(b) 1, 3 and 4 only
Which of the above given statements are
(c) 2 and 4 only
(d) 1, 2, 3 and 4
O correct?
(a) 1, 2 and 4 only
(b) 2 and 3 only
19. Which among the following given
SC
provisions requires Special Majority (c) 2, 3 and 4 only
of Parliament and Consent of States? (d) 3 and 4 only
1. Power of Parliament to amend the
Constitution and its procedure. 22. Consider the following statements
2. Extent of the executive power of the Union with respect to the Presidential
and the states. Electoral College:
GS

3. Election of the President and its manner. 1. The President is elected by an Electoral
College consisting of elected Members
4. Representation of states in Parliament.
of Parliament and Members of State
Select the correct answer using the code given Legislative Assemblies.
below: 2. Nominated Members of both Houses
(a) 1 and 2 only of the Parliament and Members of State
Legislative Councils also take part in
(b) 1, 2 and 3 only election.
(c) 1, 2 and 4 only 3. The election is held by means of single
(d) 1, 2, 3 and 4 transferable vote, system of proportional
representation.

20. Consider the following statements with 4. The voting is done by open ballot to
reduce cross voting.
respect to the Substitute Motions:
Which of the above statements are correct?
1. These are motions which, though
independent in form, are moved in the (a) 1 and 4 only
course of debate on another question (b) 2 and 3 only
and seek to supersede that question.
(c) 1 and 3 only
2. Amendments to substitute motions are
not permissible. (d) 1, 2, 3 and 4

GS SCORE Google it:- PDF4Exams


5
https://t.me/pdf4exams «Downloaded From» https://t.me/testseries4exams

23. Consider the following statements with 26. Consider the following statements
respect to the Council of Minister: related to CAG:
1. Minister of State are never given 1. Comptroller and Auditor General of India
independent charge of ministry and (CAG) controls financial system of the
always remain attached to the Cabinet country.
Ministers. 2. Finance Minister represents Office of the
2. Cabinet Minister are collectively Comptroller and Auditor General in the
responsible to Lok Sabha and has to Parliament.
resign collectively if No Confidence 3. The accounts of the Union and the
Motion passed, not complete Council of States shall be kept in form as President
Ministers.
prescribes on advice of CAG.
3. Council of Ministers are headed by the
Prime Minister. Which of the above statements is/are correct?

Which of the above statements is/are correct? (a) 1 and 2 only

(a) 1 and 2 only (b) 2 and 3 only

(b) 3 only (c) 1 and 3 only

RE
(c) 1 and 3 only (d) 1, 2 and 3

(d) 1, 2 and 3
27. Consider the following statements
related to the State Legislature:
24. Which of the following are the
1. Legislative Council of State is appointed
qualifications for Election as
for five years.
President?
O 2. Total number of members in the Legislative
1. He should be qualified for election as a Council must not exceed fifty percent of
member of the Parliament. legislative assembly.
2. He should not hold any office of profit. 3. Any citizen of India above age 25 can
SC
3. He should have completed 30 years of become member of Legislative Assembly.
age.
Which of the above statements is/are
Select the correct answer using the code given incorrect?
below:
(a) 1 and 2 only
(a) 2 only
(b) 2 and 3 only
(b) 2 and 3 only
(c) 3 only
GS

(c) 1 and 2 only


(d) 1 and 3 only
(d) 1, 2 and 3
28. With reference to the Governor
25. Consider the following statements in State, consider the following
about the Parliament of India: statements:
1. Rajya Sabha represents People of India as 1. Governor can be removed from his office
a whole. according to the procedure mentioned in
2. Lok Sabha represents States and the Article 153.
Union Territories. 2. Governor can hold office beyond the term
3. President remains member of both the of five years.
Houses of Parliament, until he resigns. 3. Governor does not possess any diplomatic
Which of the above statements is/are power.
incorrect? Which of the above statements is/are correct?
(a) 1 and 2 only (a) 1 only
(b) 2 and 3 only (b) 1 and 2 only
(c) 3 only (c) 2 and 3 only
(d) 1, 2 and 3 (d) 1 and 3 only

6
Google it:- PDF4Exams GS SCORE
https://t.me/pdf4exams «Downloaded From» https://t.me/testseries4exams

29. Consider the following statements 32. Consider the following statements
with reference to the Prime Minister with reference to the functioning of
of India: the Indian Parliament:
1. It is the Council of Minister but not Prime 1. Language of transaction in Parliament is
Minister mentioned anywhere in the English only.
Constitution. 2. Only Ministers can speak in his/her mother
2. President must act in accordance with the tongue in the Parliament.
advice tendered by the Prime Minister 3. Attorney General can speak in the Joint
always. Sitting of the Parliament.
Which of the above statements is/are correct? Which of the above statements is/are correct?
(a) 1 only (a) 1 and 2 only
(b) 2 only (b) 2 only
(c) Both 1 and 2 (c) 1 and 3 only
(d) Neither 1 nor 2 (d) 3 only

RE
30. With reference to All India Judicial 33. Consider the following statements
Services (AIJS), consider the following with reference to Attorney General of
statements: India:
1. AIJS has been added in the Indian 1. He remains the Member of Parliament
Constitution on the recommendation of
O even after the dissolution of Cabinet.
Swaran Singh Committee. 2. For the post of Attorney General, Prime
2. AIJS may help in creating more accountable Minister appoints a person who is
and equitable judiciary. qualified to become the Judge of the
Supreme Court.
Which of the above statements is/are
SC
3. In accordance with Article 76, he must
incorrect? perform duties assigned to him by the
(a) 1 only Lok Sabha.
(b) 2 only Which of the above statements is/are
incorrect?
(c) Both 1 and 2
(a) 2 only
(d) Neither 1 nor 2
GS

(b) 2 and 3 only


31. With reference to the Upper House (c) 1 and 3 only
of Indian Parliament, which of the (d) 1, 2 and 3
following statements is/are correct?
1. Only two Union territories of Delhi and 34. Consider the following statements
Puducherry have representation in Rajya with respect to the composition of the
Sabha. Lok Sabha:
2. In Upper House maximum seats are 1. All the listed Electorals can vote in Lok
occupied by representatives of the states. Sabha elections.
3. Allocation of seats to the states in Upper 2. Lok Sabha has 552 members at present.
House is on the basis of population. 3. 61st Constitutional Amendment Act has
Select the correct answer using the code given fixed voting age as 18 years.
below: Which of the above statements is/are correct?
(a) 1 and 2 only (a) 1 and 2 only
(b) 2 only (b) 2 and 3 only
(c) 2 and 3 only (c) 3 only
(d) 1, 2 and 3 (d) 1, 2 and 3

GS SCORE Google it:- PDF4Exams


7
https://t.me/pdf4exams «Downloaded From» https://t.me/testseries4exams

3. With amendment in Representation of


35. Consider the following statements: Peoples Act 2016, PIO (Person of India
1. Prime Minister is the Leader of the Lok Origin) is now qualified for post of Indian
Sabha always. President.
2. Vice President is the Chairman of the Which of the above statements is/are correct?
Rajya Sabha. (a) 1 and 2 only
Which of the above statements is/are correct? (b) 2 only
(a) 1 only
(c) 1 and 3 only
(b) 2 only
(d) None
(c) Both 1 and 2 only
(d) Neither 1 nor 2 39. Consider the following provisions for
Bills mentioned in the Constitution:
36. With reference to the appointment of 1. If an Ordinary Bill is defeated in the
the Council of Minister, which of the Lok Sabha, it may lead to resignation of
following statement is correct?

RE
government if introduced by a Minister.
(a) Prime Minister has discretion to choose 2. If Money Bill is defeated in Rajya Sabha,
his cabinet. Government must resign with immediate
(b) President has discretion to choose effect.
anybody as Prime Minister and Member Which of the above statements is/are correct?
of Council of Ministers.
(a) 1 only
O
(c) President has discretion to choose Kitchen
Cabinet. (b) 2 only

(d) Prime Minister can choose his ministers (c) Both 1 and 2
among either house of Parliament only. (d) Neither 1 nor 2
SC
37. Consider the following pairs with 40. Consider the following statements:
respect to Sessions of the Parliament:
1. The Indian Parliament is competent to
Session Description make laws on matters enumerated in the
1. Prorogation It terminates the work in Union List and concurrent list.
sitting for specified time. 2. Laws made by Parliament may extend
GS

2. Adjournment It terminates the session of throughout or in part of the economic


the House territory of India only.
3. Dissolution It ends the very life of the Which of the above statements is/are correct?
existing House
(a) 1 only
Which of the above pairs is/are incorrectly
matched? (b) 2 only
(a) 3 only (c) Both 1 and 2
(b) 1 and 2 only (d) Neither 1 nor 2
(c) 2 and 3 only
(d) 1, 2 and 3
41. Consider the following statements
with respect to the removal of
President of India:
38. Consider the following statements
related to the Presidential Election: 1. President can be removed for violation of
Constitution only.
1. All disputes related to Presidential election
in India are settled finally by the Election 2. Violation of Constitution is defined in
Commission of India. Article 61.
2. President in India is elected for five years 3. I mpeachment process must be initiated in
and maximum two terms. Lok Sabha only.

8
Google it:- PDF4Exams GS SCORE
https://t.me/pdf4exams «Downloaded From» https://t.me/testseries4exams

Which of the above statements is/are 45. With reference to the Chief Minister
incorrect? in State, consider the following
(a) 1 and 2 only statements:
(b) 2 only 1. He is de jure executive of the State.

(c) 2 and 3 only 2. No specific procedure is mentioned in


constitution for his appointment.
(d) 1 and 3 only
3. He takes oath of office and secrecy from
Judge of High court.
42. Consider the following statements
Which of the above statements is/are correct?
with respect to the Writ jurisdiction of
the Supreme Court and High Court: (a) 2 only
1. Supreme Court can issue writs to enforce (b) 1 and 2 only
both fundamental rights and legal rights. (c) 3 only
2. The High Court can issue writs throughout
(d) 1 and 3 only
the territory of India like Supreme Court.

RE
3. High Court can refuse to exercise its writ
46. Consider the following statements:
jurisdiction.
1. Union executive consists of President,
Which of the above statements is/are correct?
Prime minister and Council of Ministers
(a) 1 and 2 only only.
(b) 3 only 2. Any person who qualifies conditions
(c) 2 and 3 only
(d) 1, 2 and 3
O under Article 66 can contest election of
President.
3. Electoral College for purpose of
Presidential election includes elected
SC
43. With reference to the functions of the members of Pondicherry also.
Prime Minister, which of the following Which of the above statements is/are
statements is correct? incorrect?
(a) The salaries and allowances of Ministers (a) 1 and 2 only
are decided by him.
(b) 2 and 3 only
GS

(b) He advices the President to dismiss the


(c) 1 and 3 only
minister in case of difference of opinion.
(d) 1, 2 and 3
(c) He advices President for appointment of
Speaker.
47. Which of the following statement is
(d) He allocates fund to each Ministry.
correct regarding the appointment of
the District Judge?
44. For implementation of decisions
made at the International conference (a) District judge is appointed by Governor
of the state in consultation with the
or associations, Parliament can make
President.
law:
(b) District judge is appointed by the High
(a) With necessary ratification of minimum
Court in consultation with the Governor.
fifty percent of States.
(c) District Judge is appointed by the State
(b) Without consent of any State.
Governor on the advice of Chief Justice of
(c) With necessary consent of all the States the concerned High Court.
and Union Territories.
(d) District Judge is appointed by the Chief
(d) With necessary guidance of United Justice of the High Court in consultation
Nations. with the President.

GS SCORE Google it:- PDF4Exams


9
https://t.me/pdf4exams «Downloaded From» https://t.me/testseries4exams

48. Minister of Parliaments (MPs) may Select the correct answer using the code given
move a motion for which of the below:
following reasons? (a) 1 and 2 only
1. Discussing important issues such as
(b) 2 only
inflation, drought, and corruption.
2. Adjournment of business in a House (c) 2 and 3 only
in order to express displeasure over a (d) 1, 2 and 3
government policy.
3. Expressing no confidence in the 51. Which of the following pardoning
government leading to its resignation.
powers of the President is/are correctly
Select the correct answer using the code given matched?
below:
1. Pardon - It removes both the sentence and
(a) 1 only
the conviction and completely absolves
(b) 1 and 3 only the convict from all sentences.
(c) 1 and 2 only 2. Commutation - It denotes the substitution

RE
(d) 1, 2 and 3 of one form of punishment with a lighter
form but not in case of court martial.

49. Which of the following statements 3. Respite - It implies a stay of the execution
is/are correct regarding Attorney of sentence for a temporary period.
General of India? 4. Reprieve - It denotes awarding a lesser
1. He is a government servant who gives sentence in place of one originally
O
advice to the Government upon legal awarded due to some special factor.
matter which is referred to him by the Select the correct answer using the code given
President. below:
2. He should not defend accused person in
SC
(a) 1 only
criminal prosecution without permission
of the GOI. (b) 1, 2 and 3 only
3. He appear on behalf of the Government (c) 1, 2 and 4 only
of India in all cases in the Supreme Court
in which the Government of India is (d) 1, 2, 3 and 4
concerned.
Select the correct answer using the code given 52. The original jurisdiction of the
GS

below: Supreme Court does not cover which


(a) 2 and 3 only of the following matters?
(b) 1 and 3 only 1. Disputes arising between the Government
of India and one or more States.
(c) 2 only
2. Inter-state water disputes.
(d) 1, 2 and 3
3. Enforcement of fundamental rights.
50. Which of the following statements 4. Adjustment of certain expenses and
is/are correct regarding Lame-duck pensions between the centre and states.
session? 5. Ordinary dispute of commercial nature
1. It is conducted after election of new between the centre & the states.
members but before they are installed. Select the correct answer using the code given
2. It refers to a session in which members below:
participated for last time because of
(a) 1, 2, 3 and 4 only
failure in re-election to Lok Sabha.
3. Lame-duck session requires minimum (b) 2, 4 and 5 only
number of members whose presence is (c) 2, 3 and 5 only
essential to transact the business of the
House. (d) 1, 2, 3 and 5 only

10
Google it:- PDF4Exams GS SCORE
https://t.me/pdf4exams «Downloaded From» https://t.me/testseries4exams

53. Which of the following officials are 4. If a person is elected to both the Houses
appointed as well as removed by the of a State Legislature, he has to consult
Governor? Speaker for the procedure of vacation of
seat.
1. Advocate General of the state.
Select the correct answer using the code given
2. State Election Commissioner. below:
3. Members of the State Public Service
(a) 1 and 2 only
Commission.
4. Members of the State Human Rights (b) 1, 2 and 4 only
Commission. (c) 2, 3 and 4 only
Select the correct answer using the code given (d) 1, 2, 3 and 4
below:
(a) 1 only 56. Which of the following pairs are
(b) 1, 2 and 4 only correctly matched?
(c) 1, 3 and 4 only Articles Provisions
(d) 1, 2, 3 and 4 1. Article 74 There shall be Council of

RE
Ministers headed by P.M.
54. Which of the following statements 2. Article 75 Council of Ministers shall
be collectively responsible
are correct regarding enactment of
to L.S
the Budget?
3. Article 77 Conduct of Business of
1. Parliament can reduce or increase tax but GOI (Government of India)
cannot abolish it.

for the preparation of the budget.


O
2. The Department of revenue is responsible

3. Passing of an appropriation bill completes


4. Article 78

Duty of the Prime Minister
with respect to President.
Select the correct answer using the code given
the process of enactment of the budget. below:
SC
4. The term ‘budget’ used in the Constitution (a) 1, 2 and 3 only
has been dealt with in Article 112 of the (b) 1, 3 and 4 only
Constitution.
(c) 2, 3 and 4 only
Select the correct answer using the code given
below: (d) 1, 2, 3 and 4

(a) 1, 2 and 3 only


GS

57. Which of the following statements is/


(b) 1 and 3 only are correct regarding Zero hour and
(c) 2, 3 and 4 only calling attention motion?
(d) None 1. Zero hour is an informal device available
to members of parliament to raise matter
without any prior notice whereas calling
55. A person cannot be a member of both attention motion used to call the attention
houses of parliament at the same of minister to a matter of urgent public
time. Thus, which of the following importance.
provisions is/are correct regarding 2. Unlike calling attention motion, zero hour
vacation of seats? is mentioned in the rule of procedure.
1. If sitting member of one house is elected 3. Both are the Indian innovations in the
to the other house, his seat in the first field of parliamentary procedures.
house becomes vacant.
Select the correct answer using the code given
2. If person is elected to both the houses of
below:
parliament and in default of intimation his
seat in Rajya Sabha becomes vacant. (a) 1 only
3. If a person is elected to seat of parliament (b) 1 and 2 only
and state legislature at the same time
(c) 1 and 3 only
then his seat in state legislature becomes
vacant. (d) 1, 2 and 3

GS SCORE Google it:- PDF4Exams


11
https://t.me/pdf4exams «Downloaded From» https://t.me/testseries4exams

58. Consider the following statements Select the correct answer using the code given
related to the Acts made by below:
Parliament: (a) 1 and 3 only
1. President may repeal or amend any act (b) 1, 3 and 4 only
of Parliament in relation to Andaman and
Nicobar Islands and Lakshadweep. (c) 1 and 4 only
2. The Governor is empowered to direct that (d) 1, 2, 3 and 4
an act of Parliament does not apply to a
scheduled area in the state.
61. Which of the following roles is/are
3. The laws of the Parliament are not assigned to Vice President as Chairman
applicable to the Indian citizens and their
property in any part of the world as it of the Rajya Sabha?
lacks sovereignty. 1. He is the final authority on the
interpretation of the Constitution and the
Which of the above statements is/are correct?
Rules of Procedure for all house-related
(a) 1 and 2 only matters.
(b) 3 only 2. He determines whether a Rajya Sabha

RE
(c) 1 only member stands to be disqualified on
grounds of defection.
(d) 1, 2 and 3
3. He acts as guardian of parliamentary
privileges and his consent is required to
59. Which of the following is correct raise a question of breach of privilege.
regarding the duties and functions of
Select the incorrect answer using the codes
CAG as laid down by the Parliament
O given below:
and the Constitution?
(a) 1 and 3 only
1. He audits balance sheets and other
subsidiary accounts kept by any department (b) 1 only
SC
of central and state government.
(c) 1, 2 and 3
2. He can audit the accounts of Panchayati
Raj Institutions and Urban Local Bodies (d) None
but with the permission of the State
government. 62. Which of the following is/are the
3. He ascertains and certifies the net functions of the union executives?
proceeds of any tax or duty.
1. The initiation of legislation.
GS

4. He audits and reports on the trade


2. The promotion and implementation of
and manufacture by government
departments. welfare issues of economy and social
issues.
Select the correct answer using the code given
3. It can make laws on subjects that fall
below:
within the State List.
(a) 1, 2 and 3 only
Select the correct answer using the code given
(b) 2, 3 and 4 only below:
(c) 1, 3 and 4 only (a) 1 and 2 only
(d) 1, 2, 3 and 4 (b) 2 only
(c) 2 and 3 only
60. Supreme Court’s jurisdiction and
powers can be enlarged by which of (d) 1, 2 and 3
the following?
1. Parliament 63. Which of the following bills has/have
2. State legislature been passed as Money Bill in India?
3. President 1. Aadhaar (Targeted Delivery of Financial
and Other Subsidies, Benefits and
4. Special agreement of Centre and States. Services) Bill.

12
Google it:- PDF4Exams GS SCORE
https://t.me/pdf4exams «Downloaded From» https://t.me/testseries4exams

2. Compensatory Afforestation Fund Bill, 2015. 66. Which of the following parliamentary
3. President’s (Emoluments and) Pension Act. terms is/are correctly described?
4. Goods and Service Tax Act. 1. Whip - The office of whip is mentioned
neither in the constitution of India nor in
Select the correct answer using the code given
the rules of House nor in the parliamentary
below:
statue.
(a) 1 and 3 only
2. Crossing the floor - Passing between
(b) 1 only the member addressing the House and
(c) 1, 3 and 4 only the Chair which is considered breach of
Parliamentary etiquette.
(d) 1, 2, 3 and 4
3. Subordinate Legislation - Rules framed by
the Executive in pursuance of the power
64. Which of the following statements is/ conferred on it by the Constitution
are correct regarding the process of
Select the correct answer using the code given
election of the President of India? below:
1. The process of election of the President is
(a) 1 only
of Indian origin and no other constitution

RE
contains a similar procedure. (b) 1 and 2 only
2. Solving all the doubts and disputes (c) 2 and 3 only
in connection with the election of the
President is exclusive jurisdiction of the (d) 1, 2 and 3
Supreme Court.
Select the correct answer using the code given
O 67. Which of the following Bills require
below: the prior recommendation of the
(a) 1 only President for introduction in the
Parliament?
(b) 2 only
1. Bills related to making changes in areas of
SC
(c) Both 1 and 2 existing states.
(d) Neither 1 nor 2 2. Bills related to abolition of any tax in
which states are interested.
65. Which of the following statements are 3. A bill which would involve expenditure
correct regarding the Supreme Court from Consolidated Fund of India.
and High Court? 4. State bills imposing restrictions upon the
GS

1. Number of judges in the Supreme Court freedom of trade.


can be increased by the parliament
Select the correct answer using the code given
whereas number of judges in the High
below:
Court can be increased by the President.
2. Impeachment process of judge of the (a) 1, 2 and 4 only
Supreme Court and High court is same. (b) 1, 2 and 3 only
3. Salaries, allowances, pensions and leaves (c) 1 and 2 only
of the judges of the Supreme Court
and high court are determined by the (d) 1, 2, 3 and 4
Parliament.
4. Salaries, allowances, pensions of the 68. Which of the following statements are
judges of both are charged on the correct regarding Contingency Fund
Consolidated Fund of India. of India?
Select the correct answer using the code given 1. It aims to facilitate meeting of urgent
below: unforeseen expenditure by the
(a) 1, 2 and 3 only government without authorization from
parliament.
(b) 1, 3 and 4 only
2. Amount from Consolidated Fund of India
(c) 2, 3 and 4 only
is used to recoup the Contingency Fund
(d) 1, 2, 3 and 4 of India.

GS SCORE Google it:- PDF4Exams


13
https://t.me/pdf4exams «Downloaded From» https://t.me/testseries4exams

3. The fund is held by the finance secretary 71. Which of the following statements
on behalf of President of India. are correct regarding consultative
4. Like public account of India, it is also committee?
operated by executive action.
1. It provides a forum for informal discussion
Select the correct answer using the code given between the ministers and the members of
below: the parliament on policies and programs
(a) 1, 3 and 4 only of the government and manner of their
implementation.
(b) 2, 3 and 4 only
2. These committees are constituted by the
(c) 1, 2 and 3 only Ministry of Parliamentary Affairs.
(d) 1, 2, 3 and 4 3. These committees shall stand dissolved
upon dissolution of every Lok Sabha and
69. Which of the following statements shall be reconstituted upon constitution
is/are incorrect regarding the of each Lok Sabha.
parliamentary proceedings? Select the correct answer using the code given
1. Cut motion is a power given to the below:

RE
members of the Parliament to oppose a (a) 1 and 3 only
demand in the Financial Bill.
(b) 2 and 3 only
2. The provision of joint sitting is applicable
to ordinary bills only and not to money (c) 1 and 2 only
bills or constitutional bills.
(d) 1, 2 and 3
3. In the absence of speaker and deputy
speaker, chairman of Rajya Sabha presides
O
over a joint sitting of two Houses. 72. Which of the following statements are
Select the correct answer using the code given
correct regarding Lok Adalats?
below: 1. It is presided over by a sitting or retired
judicial officer as the Chairman.
SC
(a) 3 only
2. Its decision is binding on parties to
(b) 2 only
the dispute and its order is capable of
(c) 2 and 3 only execution through legal process.
(d) 1, 2 and 3 3. It deals in matters related to both civil and
criminal cases.
70. Which of the following statements are 4. If the case is already filed in the regular
GS

correct regarding executive powers of court, the fee paid will be refunded if the
the President? dispute is settled at Lok Adalats.
1. All executive actions of the Government Select the correct answer using the code given
of India are formally taken in his name. below:
2. He can make rules for more convenient (a) 1, 2 and 3 only
transaction of business of Union
government. (b) 2, 3 and 4 only
3. He can preside over an inter-state council (c) 1, 3 and 4 only
to promote centre-state and inter-state (d) 1, 2, 3 and 4
cooperation.
4. He can make regulations for peace,
progress and good governance of the 73. Which of the following statements is/
Daman and Diu. are correct regarding post of Leader
Select the correct answer using the code given
of Opposition?
below: 1. Post of Leader of Opposition is given to
Opposition party having not less than
(a) 1 and 2 only
one-tenth seats of the total strength of
(b) 1, 2 and 3 only the House
(c) 1, 3 and 4 only 2. Original constitution provided statutory
(d) 1, 2, 3 and 4 recognition to the leader of opposition.

14
Google it:- PDF4Exams GS SCORE
https://t.me/pdf4exams «Downloaded From» https://t.me/testseries4exams

3. He is entitled to salary, allowances and 76. Consider the following statements


other facilities equivalent to that of a about powers of the President of
cabinet minister. India:
Select the correct answer using the code given 1. He appoints Attorney General of India
below: and Comptroller & Auditor General of
India who hold office during the pleasure
(a) 1 and 3 only of the President.
(b) 2 and 3 only 2. He can address Parliament at the
(c) 3 only commencement of the first session after
each general election and first session of
(d) 1, 2 and 3 each financial year.
3. He constitutes a Finance Commission
74. Which of the following statements are after every five years to recommend the
correct regarding ordinance making distribution of revenues between the
centre and states.
power of the President?
Which of the above statements is/are correct?
1. President can make ordinance without

RE
the instruction from Council of Ministers (a) 1 and 3 only
headed by PM. (b) 1 and 2 only
2. Ordinance making power of the President (c) 3 only
to legislate is a parallel power of
(d) 1, 2 and 3 only
legislation. O
3. The maximum life of an ordinance can be
77. Consider the following statements
six months only.
with respect to responsibilities of
Select the correct answer using the code given Governor of Arunachal Pradesh under
below: Article 371-H:
SC
(a) 1 and 2 only 1. Governor exercises his individual
judgment which is independent of Council
(b) 2 and 3 only
of Ministers.
(c) 1, 2 and 3 2. This special responsibility of Governor
(d) None ceases when the President so directs.
3. Governor should establish a Regional
GS

Council for the Tuensang district and make


75. Consider the following statements
rules for the compositions and service
about Ad hoc judge: conditions of the member of council.
1. He is appointed by the Chief Justice of Which of the above statements is/are correct?
India when there is a lack of quorum of
the permanent judges to hold or continue (a) 2 only
any session of the Supreme Court. (b) 1 and 3 only
2. He can do so only with the previous (c) 2 and 3 only
consent of the President. (d) 1, 2 and 3
3. He shall have all the jurisdiction, powers
and privileges of a Judge of the Supreme 78. Consider the following statements
Court. with respect to Territorial Extent of
Which of the above statements are correct? Central and State Legislation:
(a) 1 and 3 only 1. The Parliament can make laws for the
whole or any part of the territory of
(b) 2 and 3 only India.
(c) 1 and 2 only 2. A state legislature can make laws for the
whole or any part of the state.
(d) 1, 2 and 3

GS SCORE Google it:- PDF4Exams


15
https://t.me/pdf4exams «Downloaded From» https://t.me/testseries4exams

3. The Parliament can also make extra- 81. Which of the following statements
territorial legislation. related to the provisions mentioned
Which of the above statements is/are correct? in the Constitution is/are correct?
(a) 1 and 2 only 1. All doubts and disputes in connection
with election of the Vice-President are
(b) 2 only
inquired into and decided by the Election
(c) 1 and 3 only Commission whose decision is final.
(d) 1, 2 and 3 2. The Prime Minister shall be appointed by
the President and the other ministers shall
be appointed by the Prime Minister.
79. Which of the following statements is/
are correct regarding language of the Select the correct answer using the code given
judiciary and texts of Laws? below:

1. Hindi translation of acts, ordinances and (a) 1 only


bye laws published under the authority (b) 2 only

RE
of the President are deemed to be
authoritative text. (c) Both 1 and 2

2. President can authorize the use of Hindi (d) Neither 1 nor 2


for the judgment, decrees and orders
passed by the high court of the state. 82. Consider the following statements
3. Supreme Court hears only those who related to the Vice President:
O
petition or appeal in either Hindi or 1. The Chairman of the Rajya Sabha can
English. be removed from his office only if he is
Select the correct answer using the code given removed from the office of the Vice-
below: President.
SC
(a) 1 only 2. The Vice President at the time of his
removal can be present and speak in the
(b) 2 only House and can take part in its proceedings,
(c) 1 and 3 only without voting.

(d) 1, 2 and 3 3. The Vice-President cannot preside over a


sitting of the Rajya Sabha as its Chairman,
GS

when a resolution for his removal is under


80. Consider the following statements consideration.
regarding the State’s consent with
Which of the above statements is/are correct?
respect to bills;
1. There is six months’ time limit for the (a) 1 and 2 only
states to give consent in those provisions (b) 1 only
of the Constitution which are related to
(c) 1, 2 and 3
federal structure of polity.
(d) None
2. Bills related to the formation of new state
or changes in the existing states of union
of India have no time limit for consent of 83. Which of the following dispute fall
states. under the Original Jurisdiction of
Which of the above statements is/are correct? Supreme Court?
(a) 1 only 1. Ordinary dispute of Commercial nature
between the Centre and the States.
(b) 2 only
2. Inter-state water disputes.
(c) Both 1 and 2
3. Recovery of damages by a State against
(d) Neither 1 nor 2 the Centre.

16
Google it:- PDF4Exams GS SCORE
https://t.me/pdf4exams «Downloaded From» https://t.me/testseries4exams

Select the correct answer using the code given 86. In a representative democracy,
below: elected representatives make laws on
(a) 2 only behalf of citizens. Citizens’ ability to
(b) 3 only participate in the legislative process is
fundamental to democracy. Consider
(c) 1 and 3 only
the following statements in this
(d) None context:
1. Legislation may be initiated by stakeholders
84. In context of the power provided by or from within the government.
the Constitution to the Parliament to
2. Public participation regarding draft Bills is
reorganize the states, consider the statutorily mandated in India.
following statements:
Which of the above statements is/are correct?
1. In case of Union Territory reorganization,
reference need to be made to concerned (a) 1 only
legislature to ascertain its views. (b) 2 only

RE
2. Views of the concerned state legislature/ (c) Both 1 and 2
UT are binding on the President.
(d) Neither 1 nor 2
3. Such laws does not require a special
majority but can be passed by a simple
majority. 87. Which of the following statements
Which of the above statements is/are correct?
O about financial powers and functions
of the President are correct?
(a) 1 and 2 only
1. He can make advances out of the
(b) 2 and 3 only Consolidated Fund of India to meet any
SC
(c) 3 only unforeseen expenditure.

(d) 1, 2 and 3 2. No demand for a grant can be made


except on his recommendation.

85. Regarding the Office of the President 3. Money bills can be introduced in
the Parliament only with his prior
of India which of the following
recommendation.
statements is/are correct?
GS

Select the correct answer using the codes


1. His electoral college consist of all members
given below:
of both the Houses of Parliament, elected
members of Legislative Assembly of (a) 1 and 2 only
States and UT.
(b) 2 and 3 only
2. He should be qualified for election as a
(c) 1 and 3 only
member of the Lok Sabha only.
3. His oath of office is administered by the (d) 1, 2 and 3
Vice-President of India.
4. He can resign from office by addressing 88. Consider the following statements
the resignation letter to the Chief Justice about the features of Cabinet
of India. Committees:
Select the correct answer using the code given 1. They are constitutional in nature.
below: 2. They not only include the Ministers in
(a) 1 and 2 only charge of subjects covered by them but
also include other senior Ministers.
(b) 1 and 3 only
3. They not only sort out issues and
(c) 2 only
formulate proposals for the consideration
(d) None of the Cabinet, but also take decisions.

GS SCORE Google it:- PDF4Exams


17
https://t.me/pdf4exams «Downloaded From» https://t.me/testseries4exams

Which of the above statements is/are correct? 91. The role of CAG is to uphold the
(a) 1 and 2 only Constitution of India and the laws of
Parliament in the field of financial
(b) 2 and 3 only
administration. Which of the
(c) 1 and 3 only following statements in this context
(d) 1, 2 and 3 are correct?
1. The CAG has more freedom with regard
89. The Supreme Court has examined the to audit of expenditure than with regard
pardoning power of the President to audit of receipts, stores and stock.

under different cases and laid down 2. In addition to this legal and regulatory
audit, the CAG cannot conduct the
various principles. Which of the
propriety audit.
following statements in this context
3. The CAG has no control over the issue of
are correct?
money from the Consolidated Fund of
1. The petitioner for mercy has no right to India.

RE
an oral hearing by the President.
Select the correct answer using the code given
2. The power is to be exercised by the below:
President on the advice of the Union (a) 1 and 2 only
Cabinet.
(b) 2 and 3 only
3. The President is not bound to give reasons
for his order.
O (c) 1 and 3 only
(d) 1, 2 and 3
Select the correct answer using the code given
below:
SC
92. Consider the following statements
(a) 1 and 2 only
in context with the adjournment
(b) 2 and 3 only motion:
(c) 1 and 3 only 1. It should raise a matter of urgent public
importance.
(d) 1, 2 and 3
2. This can be admitted both in Lok Sabha
GS

as well as Rajya Sabha.


90. Consider the following statements
Which of the above statements is/are correct:
about powers and functions of Prime
(a) 1 only
Minister:
(b) 2 only
1. He can ask a minister to resign or advise
the President to dismiss him in case of (c) Both 1 and 2
difference of opinion. (d) Neither 1 nor 2
2. He can recommend dissolution of the Lok
Sabha to the President at any time. 93. Consider the following statements
3. He is the crisis manager-in-chief at the related to the classification of Council
political level during emergencies. of Ministers:
Which of the above statements are correct? 1. The Cabinet is a smaller body which
consists of a few important senior
(a) 1 and 2 only ministers who are in charge of separate
(b) 2 and 3 only departments.
2. A Deputy Minister is the junior member
(c) 1 and 3 only
to Cabinet Minister and look after their
(d) 1, 2 and 3 work in their absence.

18
Google it:- PDF4Exams GS SCORE
https://t.me/pdf4exams «Downloaded From» https://t.me/testseries4exams

3. Minister of State has an independent 96. Consider the following statements:


charge of the ministry and is a part of the 1. A person who is not a member of either
Cabinet. House of the Parliament can be appointed
Which of the above statements is/are correct? a member of the Council of Minister.
2. A minister who is not a member of either
(a) 1 only
House of the Parliament can continue to
(b) 2 only do so for 1 year.
(c) 1 and 2 only 3. A minister who is a member of one House
of Parliament has the right to speak and
(d) 1 and 3 only to take part in the proceedings of the
other House also.
94. Consider the following statements Which of the above statements is/are correct?
related to the concept of collective (a) 1, 2 and 3
responsibility of the Council of
(b) 1 only
Minister:
(c) 1 and 2 only

RE
1. The Council of Minister is collectively
responsible to the Parliament. (d) 1 and 3 only
2. A vote of no-confidence against any
minister in Rajya Sabha automatically 97. When Lok Sabha passes no confidence
leads to the resignation of the entire motion against the Council of Minister,
council. then which of the following conditions
take place?
O
3. A minister is not expected to speak out
publicly against a policy decision of the
Cabinet.
1. The ministers who are from Lok Sabha
have to resign.

Which of the above statements is/are correct? 2. Prime Minister has to resign.
SC
3. The ministers from Rajya Sabha have to
(a) 1 only
resign.
(b) 1 and 2 only 4. The President may not oblige the Council
(c) 3 only of Minister who has lost confidence of the
Lok Sabha.
(d) 2 and 3 only
Select the correct answer using the codes
GS

given below:
95. Consider the following statements
(a) 1 and 2 only
related to the Attorney General of
India: (b) 1, 2 and 3 only
1. He is the Indian government’s chief legal (c) 1, 2 and 4 only
advisor, and is primary lawyer in the (d) 1, 2, 3 and 4
Supreme Court of India.
2. He has the right to participate in the 98. The individual privileges enjoyed
proceedings of the Parliament.
by the Members of the Parliament
3. The Attorney General is to be consulted include:
in legal matters of real importance and
1. Absolute freedom of speech.
only after the Ministry of Law has been
consulted for it. 2. They are exempted from jury service.
3. They cannot be arrested during the session
Which of the above statements are correct?
of the Parliament and 40 days before the
(a) 1 and 2 only beginning and 40 days after the end of
the session.
(b) 2 and 3 only
4. The privilege of not being arrested is also
(c) 1 and 3 only
available in case of preventive detention
(d) 1, 2 and 3 cases.

GS SCORE Google it:- PDF4Exams


19
https://t.me/pdf4exams «Downloaded From» https://t.me/testseries4exams

Which of the above stated privileges are 100. Consider the following statements
correct? related to the Rajya Sabha:
(a) 2 and 3 only
1. The representatives of states in the Rajya
(b) 1 and 2 only Sabha are elected by the elected members
(c) 2 only of state legislative assemblies.
(d) 1, 2, 3 and 4 2. Seats are allocated to the states on the
basis of population of the state.
99. The question of disqualification of the 3. A member of the Rajya Sabha must be a
Members of the Parliament, according domicile of the state from which he/she
to Representation of Peoples Act 1951, is elected.
has to be decided by:
4. Union Territories have no representation
(a) Election Commission, in consultation with
in Rajya Sabha.
the Speaker and Chairman of the House.
(b) Speaker and Chairman of the House, Which of the above statements is/are correct?

RE
in consultation with the Election (a) 1, 2 and 3 only
Commission.
(b) 1 and 2 only
(c) The President, in consultation with the
Election Commission. (c) 2 and 3 only
(d) Supreme Court (d) 1, 2 and 4 only
O ***********
SC
GS

20
Google it:- PDF4Exams GS SCORE
https://t.me/pdf4exams «Downloaded From» https://t.me/testseries4exams

GS SCORE 11
IAS Prelims - 2019

POLITY
(EXECUTIVE + LEGISLATURE + JUDICIARY)

RE
Answer Key
Q. 1 (d) Q. 26 (c) Q. 51 (a) Q. 76 (c)
Q. 2 (b) Q. 27 (a) Q. 52 (b) Q. 77 (a)
Q. 3 (a) Q. 28 (c) O Q. 53 (a) Q. 78 (d)
Q. 4 (b) Q. 29 (d) Q. 54 (d) Q. 79 (a)
Q. 5 (a) Q. 30 (d) Q. 55 (a) Q. 80 (d)
Q. 6 (d) Q. 31 (d) Q. 56 (d) Q. 81 (d)
SC
Q. 7 (b) Q. 32 (d) Q. 57 (c) Q. 82 (c)
Q. 8 (c) Q. 33 (d) Q. 58 (a) Q. 83 (d)
Q. 9 (c) Q. 34 (c) Q. 59 (d) Q. 84 (c)
Q. 10 (b) Q. 35 (b) Q. 60 (c) Q. 85 (c)
Q. 11 (a) Q. 36 (a) Q. 61 (d) Q. 86 (a)
Q. 12 (b) Q. 37 (b) Q. 62 (a) Q. 87 (b)
GS

Q. 13 (b) Q. 38 (d) Q. 63 (b) Q. 88 (b)


Q. 14 (a) Q. 39 (a) Q. 64 (c) Q. 89 (d)
Q. 15 (b) Q. 40 (a) Q. 65 (a) Q. 90 (d)
Q. 16 (d) Q. 41 (c) Q. 66 (d) Q. 91 (c)
Q. 17 (d) Q. 42 (b) Q. 67 (d) Q. 92 (a)
Q. 18 (d) Q. 43 (b) Q. 68 (b) Q. 93 (a)
Q. 19 (d) Q. 44 (b) Q. 69 (a) Q. 94 (c)
Q. 20 (b) Q. 45 (a) Q. 70 (a) Q. 95 (d)
Q. 21 (b) Q. 46 (a) Q. 71 (d) Q. 96 (d)
Q. 22 (c) Q. 47 (c) Q. 72 (d) Q. 97 (d)
Q. 23 (b) Q. 48 (d) Q. 73 (a) Q. 98 (a)
Q. 24 (a) Q. 49 (a) Q. 74 (d) Q. 99 (c)
Q. 25 (d) Q. 50 (a) Q. 75 (d) Q. 100 (b)

DO NOT OPEN THIS TEST BOOKLET UNTIL YOU ARE ASKED TO DO SO

GS SCORE Google it:- PDF4Exams


1
PTS2019/B-1/102018/11
https://t.me/pdf4exams «Downloaded From» https://t.me/testseries4exams

1. Correct Option: (d) same system in the case of Lok Sabha.


The system of proportional representation
Explanation:
by means of the single transferable vote
Powers of the Parliament: used in the elections of President, Vice
President, Members of Rajya Sabha and
! Statement 2, 3, 4 and 5 are correct:
the members of state legislative council.
Following are the judicial powers of
Parliament.  Statement 2 is correct: In Lok Sabha
 It can impeach the President for elections, one member from each territory
the violation of the Constitution. get elected through the First past the
However, the Constitution does not post system. The candidate who wins
define the meaning of the phrase maximum number of votes get elected.
‘violation of the Constitution’. The  A first-past-the-post (FPTP) voting method
impeachment resolution must be is one in which voters indicate on a ballot
passed by a majority of two-thirds the candidate of their choice, and the
of the total membership in both the candidate who receives the most votes
houses separately. The impeachment wins. This is described as winner takes
is a quasi-judicial procedure in the all. First-past-the-post voting is a plurality

RE
Parliament. voting method.
 It can remove the Vice-President from  Statement 3 is incorrect: It is the first-
his office. A formal impeachment past-the-post (FPTP) voting method which
is not required for his removal. He does not secure the due representation
can be removed by a resolution of the
to minorities.
Rajya Sabha passed by an absolute
majority (i.e. , a majority of the total  The Proportional representation system
members of the House) and agreed to
O characterizes electoral systems by which
by the Lok Sabha. divisions in an electorate are reflected
proportionately in the elected body. If
 It can recommend the removal of
n% of the electorate supports a particular
judges (including chief justice) of
SC
the Supreme Court and the high political party, then roughly n% of seats
courts, chief election commissioner, will be won by that party.
comptroller and auditor general to the  The essence of such systems is that all
president. votes contribute to the result and not just
 It can punish its members or outsiders a plurality, or a bare majority, of them.
for the breach of its privileges or its So it provides the due representation to
contempt. minorities.
GS

 Statement 1 is incorrect: Power to


make laws to regulate the elections 3. Correct Option: (a)
to the offices of President and Vice- Explanation:
President, is electoral power and
function of the Parliament. Different types of majority:
 Statement 1 is correct: Special majority
2. Correct Option: (b) is the majority that is, more than 50 per
cent of the total membership of each
Explanation:
House and a majority of not less than
System of Representation in the two-thirds of the members present and
Parliament: voting in each House.
In Indian parliamentary system there are  The provisions which can be amended by
two types of election systems, the system of this majority are:
proportional representation by means of the
single transferable vote and the First past the  Fundamental Rights;
post system.  Directive Principles of State Policy; and
 Statement 1 is incorrect: The Constitution  All other provisions which are not
has adopted the system of proportional covered under the Simple majority and
representation in the case of Rajya special majority but with the consent
Sabha, though it has not preferred the of half of all the state legislatures.

2
Google it:- PDF4Exams GS SCORE
https://t.me/pdf4exams «Downloaded From» https://t.me/testseries4exams

 Statement 2 is incorrect: The expression  CAG can also be removed by the President
‘total membership’ means the total on the same grounds and in the same
number of members comprising the manner as a judge of the Supreme
House irrespective of the fact whether Court. Means he can be removed by the
there are vacancies or absentees. President on the basis of a resolution
passed to that effect by both the Houses
 Effective Majority of the house means
of Parliament with special majority, either
more than 50% of the effective strength
on the ground of proved misbehaviour
of the house. This implies that out of
or incapacity.
the total strength, we do not take into
account members who are absent or
those seats which are vacant. When Indian 5. Correct Option: (a)
Constitution mentions “all the then Explanation:
members”, then it refers to the effective
majority. The following are removed using CAG:
effective majority. The Constitution has made the following
 Statement 3 is incorrect: Removal of provisions to safeguard and ensure the
Vice-president in Rajya Sabha. independence of CAG:

RE
 Removal of Speaker and Deputy  Statement 1 is correct: He is not eligible
Speaker of Lok Sabha and State for further office, either under the
Legislative Assembly. Government of India or of any state, after
he ceases to hold his office.

4. Correct Option: (b)  Statement 2 is incorrect: His salary and


other service conditions are determined
Explanation:
Types of Majority:
O by the Parliament. His salary is equal to
that of a judge of the Supreme Court. The
administrative expenses of the office of
! Pair 1 is incorrectly matched: the CAG, including all salaries, allowances
SC
 Vice-President may be removed from and pensions of persons serving in that
office are charged upon the Consolidated
his office by a resolution of Rajya
Fund of India.
Sabha passed by a majority of all the
then members of the Rajya Sabha Other Provisions:
and agreed to Lok Sabha. This is an
 He is provided with the security of tenure.
example of effective majority in
He can be removed by the President
Rajya Sabha.
GS

only in accordance with the procedure


! Pairs 2, 3 and 4 are correctly mentioned in the Constitution. Thus, he
matched: does not hold his office till the pleasure
of the President, though he is appointed
 Lok Sabha Speaker can be removed by
by him.
a resolution passed by a majority of all
the members of the Lok Sabha. Such  Neither his salary nor his rights in
a resolution can be moved only after respect of leave of absence, pension or
giving 14 days’ advance notice. age of retirement can be altered to his
disadvantage after his ap7pointment.
 Motion of Thanks: The first session
after each general election and the first  The conditions of service of persons
session of every fiscal year is addressed serving in the Indian Audit and Accounts
by the president. In this address, the Department and the administrative
President outlines the policies and powers of the CAG are prescribed by
programmes of the government in the the President after consultation with the
preceding year and ensuing year. At the CAG.
end of the discussion, the motion is put  No minister can represent the CAG
to vote. This motion must be passed in Parliament (both Houses) and no
in the House. Otherwise, it amounts minister can be called upon to take any
to the defeat of the government. It is responsibility for any actions done by
passed by simple majority. him.

GS SCORE Google it:- PDF4Exams


3
https://t.me/pdf4exams «Downloaded From» https://t.me/testseries4exams

6. Correct Option: (d) policies and actions. If it is passed in the


Lok Sabha, the council of ministers need
Explanation:
not resign from the office
Rajya Sabha:
 Statement 1 is correct: The Rajya Sabha 8. Correct Option: (c)
(first constituted in 1952) is a continuing Explanation:
chamber, means it is a permanent body and
not subject to dissolution. However, one- Judiciary:
third of its members retire every second  Statement 1 is incorrect: Every law
year. Their seats are filled up by fresh
passed by the Parliament has no need
elections and presidential nominations at
to approve by judiciary. Law making is a
the beginning of every third year.
power of legislative. If there is violation
 Statement 2 is incorrect: The of fundamental rights of citizens then
Constitution has not fixed the term of judiciary can interfere in legislative power.
office of members of the Rajya Sabha and Ex: Supreme Court strike down NJAC act.
left it to the Parliament. Accordingly, the
 Statement 2 is correct: It has power to
Parliament in the Representation of the

RE
review its own judgement or order. Thus,
People Act (1951) provided that the term
it is not bound by its previous decision
of office of a member of the Rajya Sabha
and can depart from it in the interest
shall be six years.
of justice or community welfare. So the
 Statement 3 is incorrect: The act also Supreme Court is a self-correcting agency.
empowered the President of India to For example, in the Kesavananda Bharati
curtail the term of members chosen in the case (1973), the Supreme Court departed
O
first Rajya Sabha. In the first batch, the from its previous judgment in the Golak
retirement of one-third members were Nath case (1967).
decided by lottery system.
 Statement 3 is correct: Any citizen can
SC
 approach to the court of law if her/his
rights are violated.
7. Correct Option: (b)
Explanation: 9. Correct Option: (c)
Collective Responsibility: Explanation:

 Statement 1 is correct: The fundamental Public Interest Litigation (PIL):


GS

principle underlying the working of  The introduction of PIL in India was


parliamentary system of government is facilitated by the relaxation of the
the principle of collective responsibility. traditional rule of ‘locus standi’.
Article 75 clearly states that the council According to this rule, only that person
of ministers is collectively responsible whose rights are infringed alone can move
to the Lok Sabha. This means that all the the court for the remedies, whereas, the
ministers own joint responsibility to the PIL is an exception to this traditional rule.
Lok Sabha for all their acts of omission In PIL, any member of the public having
and commission. They work as a team ‘sufficient interest’ can approach the court
and swim or sink together. for enforcing the rights of other persons
 Statement 2 is incorrect: When the Lok and redressal of a common grievance.
Sabha passes a no-confidence motion  Statement 1 is incorrect: The judiciary,
against the council of ministers, all the including Supreme Court, entertained
ministers have to resign including those litigation from those parties that were
ministers who are from the Rajya Sabha.
affected directly or indirectly by it. It
 Censure Motion: It can be moved means that even people, who are not
against an individual minister or a directly involved in the case, may bring to
group of ministers or the entire council the notice of the Court matters of public
of ministers. It is moved for censuring interest. Courts can also take Suo Motu
the council of ministers for specific cognizance of the matter.

4
Google it:- PDF4Exams GS SCORE
https://t.me/pdf4exams «Downloaded From» https://t.me/testseries4exams

 The cases falling under the following  The Court should also ensure that
categories will not be entertained as PIL: the petition filed by busybodies for
extraneous and ulterior motives must
 Statement 2 is incorrect: Landlord–
be discouraged by imposing exemplary
tenant matters
costs or adopting similar novel
 Service matter and those pertaining to methods to curb frivolous petitions
pension and gratuity and the petitions filed for extraneous
considerations.
 Complaints against Central/ State
Government departments and Local
Bodies. 10. Correct Option: (b)
 Admission to medical and other Explanation:
educational institution Money Bill:
 Petitions for early hearing of  Statement 1 is correct: Article 110 of
cases pending in High Courts and the Constitution deals with the definition
Subordinate Courts of money bills. It states that a bill is
Supreme Court laid down the following deemed to be a money bill if it contains

RE

guidelines for checking the misuse of the ‘only’ provisions dealing with all or any of
PIL: the following matters:

 The court must encourage genuine  The imposition, abolition, remission,


and bona fide PIL and effectively alteration or regulation of any tax;
discourage and curb the PIL field for  The regulation of the borrowing of
extraneous considerations. money by the Union government;
O
 Instead of every individual Judge
devising his own procedure for dealing
 The custody of the Consolidated Fund
of India or the contingency fund of
India, the payment of moneys into or
with PIL, it would be appropriate for
the withdrawal of money from any
SC
each High Court to properly formulate
rules for encouraging the genuine PIL such fund;
filed and discouraging PIL filed with  The appropriation of money out of the
oblique motives. Consolidated Fund of India;
 The Court should prima facie verify  Declaration of any expenditure
the credentials of the petitioner before charged on the Consolidated Fund of
entertaining the PIL. India or increasing the amount of any
GS

 The Court shall be prima facie satisfied such expenditure;


regarding the correctness of the  The receipt of money on account of
contents of petition before entertaining the Consolidated Fund of India or the
the PIL. public account of India or the custody
or issue of such money, or the audit
 Statement 3 is correct: The Court
of the accounts of the Union or of a
should be fully satisfied that
state; or
substantial public interest is involved
before entertaining the petition.  Any matter incidental to any of the
matters specified above.
 The Court should ensure that the
petition which involves larger public  Statement 2 is incorrect: If any question
interest, gravity and urgency must be arises whether a bill is a money bill or
given priority over other petitions. not, the decision of the Speaker of the
Lok Sabha is final. His decision in this
 The Court before entertaining the regard cannot be questioned in any court
PIL must ensure that the PIL is aimed of law or in the either House of Parliament
at redressal of genuine public harm or even the president. When a money
and public injury. The Court should bill is transmitted to the Rajya Sabha for
also ensure that there is no personal recommendation and presented to the
gain, private motive or oblique motive president for assent, the Speaker endorses
behind filing PIL. it as a money bill.

GS SCORE Google it:- PDF4Exams


5
https://t.me/pdf4exams «Downloaded From» https://t.me/testseries4exams

 Statement 3 is incorrect: A money bill  He must not have been convicted for
can only be introduced in the Lok Sabha any offence resulting in imprisonment
and that too on the recommendation of for two or more years. But, the detention
the President. After a money bill is passed of a person under a preventive
by the Lok Sabha, it is transmitted to the detention law is not a disqualification.
Rajya Sabha for its consideration. The  He must not have failed to lodge an
Rajya Sabha has restricted powers with account of his election expenses within
regard to a money bill. It cannot reject the time.
or amend a money bill. It can only make
 He must not have any interest in
the recommendations. It must return
government contracts, works or
the bill to the Lok Sabha within 14 days, services.
wither with or without recommendations.
The Lok Sabha can either accept or reject  He must not be a director or managing
all or any of the recommendations of the agent nor hold an office of profit in a
Rajya Sabha. corporation in which the government
has at least 25 per cent share.
 Statement 4 is correct: Every such bill is
 He must not have been dismissed from
considered to be a government bill and
government service for corruption or

RE
can be introduced only by a minister.
disloyalty to the state.
 He must not have been convicted for
11. Correct Option: (a) promoting enmity between different
Explanation: groups or for the offence of bribery.
Grounds of disqualification for being a  He must not have been punished for
member of the legislative assembly or preaching and practicing social crimes
O such as untouchability, dowry and
legislative council of a state:
sati.
Statements 1, 2 and 3 are correct.
 On the question whether a member has
Under the Constitution, a person shall become subject to any of the above
SC

be disqualified for being chosen as and disqualifications, the governor’s decision
for being a member of the legislative is final. However, he should obtain the
assembly or legislative council of a state: opinion of the Election Commission and
act accordingly.
 If he holds any office of profit under
the Union or state government  Statement 4 is incorrect: He must be not
(except that of a minister or any other less than 30 years of age in the case
office exempted by state legislature), of the legislative council and not less
GS

than 25 years of age in the case of the


 if he is of unsound mind and stands so legislative assembly.
declared by a court,
 if he is an undischarged insolvent, 12. Correct Option: (b)
 if he is not a citizen of India or has Explanation:
voluntarily acquired the citizenship
Important Parliamentary terms:
of a foreign state or is under any
acknowledgement of allegiance to a Pairs 1 and 3 are correctly matched.
foreign state, and
 Expunction: Deletion of words, phrases
 if he is so disqualified under any law or expressions from the proceedings
made by Parliament. or records of Rajya Sabha by an order
of the Chairman for being defamatory
 Accordingly, the Parliament has prescribed or indecent or unparliamentary or
a number of additional disqualifications undignified.
in the Representation of People Act
(1951). These are similar to those for  Casting Vote: The vote casted by the
Parliament. These are mentioned here: Chairman, or a Member acting as such
in the House and by the Chairman or a
 He must not have been found guilty Member acting as such in a Committee,
of certain election offences or corrupt in the case of an equality of votes on a
practices in the elections. matter.

6
Google it:- PDF4Exams GS SCORE
https://t.me/pdf4exams «Downloaded From» https://t.me/testseries4exams

 Pair 2 is incorrect: Adjournment sine information about their proceedings,


die: Termination of a sitting of the House policies and decisions. They take the
without any definite date being fixed for oath of secrecy before entering their
the next sitting.
office. The oath of secrecy to the
 Draw of lot: A method applied to ministers is administered by the
determine the relative precedence of President.
private members Bills and Resolutions,
notices of questions, half-an-hour  In his oath of secrecy, the Prime
discussions or any other notice given by Minister/Minister swears that
more than one member simultaneously he will not directly or indirectly
for being taken up on the same day. communicate or reveal to any
 Naming a Member: The drawing of person(s) any matter that is brought
attention of the House by the Chairman to under his consideration or becomes
the conduct of a member who disregards known to him as a Union Minister
the authority of the Chair or abuses the
except as may be required for the
Rules of the House by persistently and
willfully obstructing the business thereof,
due discharge of his duties as such

RE
with a view to action being taken to minister.
suspend him from the service of the House
for a period not exceeding the remainder 14. Correct Option: (a)
of the session.
Explanation:

13. Correct Option: (b)


O Financial Powers of the President:
The financial powers and functions of the
Explanation:
President are:
Form of Oath of Secrecy:
 Statement 1 is correct: Money bills can
SC
 “I, A.B.C, swear in the name of god be introduced in the Parliament only with
of solemnly affirm that I will not his prior recommendation.
directly or indirectly communicate or  Statement 2 is correct: He causes to be laid
reveal to any person or persons any before the Parliament the annual financial
matter which shall be brought under statement (ie, the Union Budget).
my consideration or shall become  No demand for a grant can be made
known to me as a Minister for the except on his recommendation.
GS

State of … except as may be required  Statement 3 is incorrect: He can make


for the due discharge of my duties advances out of the Contingency
as such Minister.” fund of India to meet any unforeseen
expenditure.
 Statement 1 is incorrect: The oath
of Secrecy in Indian Parliamentary  He constitutes a finance commission
after every five years to recommend the
system is administered only to the
distribution of revenues between the
Ministers and not to the elected Centre and the states.
MPs.
 Statement 2 is incorrect: The 15. Correct Option: (b)
ministers are constitutionally obliged Explanation:
to take the oath of secrecy before
entering their office. They cannot Speaker Pro-Tem:
hold the office without taking oath  Statement 1 is incorrect: As provided by
of Secrecy. the Constitution, the Speaker of the last
Lok Sabha vacates his office immediately
 Statement 3 is correct: The ministers before the first meeting of the newly-
operate on the principle of secrecy elected Lok Sabha. Therefore, the
of procedure and cannot divulge President appoints a member of the Lok

GS SCORE Google it:- PDF4Exams


7
https://t.me/pdf4exams «Downloaded From» https://t.me/testseries4exams

Sabha as the Speaker Pro Tem. Usually, the bill is known as the pocket veto. The
the senior most member is selected for President can exercise this veto power as
this. The President himself administers the Constitution does not prescribe any
oath to the Speaker Pro Tem. time-limit within which he has to take the
 Statement 2 is correct: The Speaker Pro decision with respect to a bill presented
Tem has all the powers of the Speaker. to him for his assent.
He presides over the first sitting of the
newly-elected Lok Sabha. 17. Correct Option: (d)
 Statement 3 is incorrect: His main Explanation:
duty is to administer oath to the new
members. He also enables the House to Comptroller and Auditor General of
elect the new Speaker. India:
 When the new Speaker is elected by the The Constitution of India (Article 148)
House, the office of the Speaker Pro Tem provides for an independent office of the
ceases to exist. Hence, this office is a Comptroller and Auditor General of India
temporary office, existing for a few days. (CAG). He is the head of the Indian Audit and
Accounts Department. He is the guardian

RE
of the public purse and controls the entire
16. Correct Option: (d) financial system of the country at both the
Explanation: level of the Centre and the state.

President and Council of Ministers.  Statement 1 is incorrect: The


Constitution (Article 149) authorises
 As per Article 74 President is bound by the the Parliament to prescribe the duties
O
advice of the council of ministers headed and powers of the CAG in relation to
by the Prime Minister. Though he has no the accounts of the Union and of the
constitutional discretion, he has some states and of any other authority or body.
situational discretion. In other words, the Accordingly, the Parliament enacted the
SC
President can act on his discretion (that is, CAG’s (Duties, Powers and Conditions of
without the advice of the ministers) under Service) act, 1971. This Act was amended
the following situations: in 1976 to separate accounts from audit
in the Central government.
 Statement 1 is correct: Appointment
of Prime Minister when no party has  Statement 2 is incorrect: He compiles
a clear majority in the Lok Sabha or and maintains the accounts of state
when the Prime Minister in office governments. In 1976, he was relieved
GS

dies suddenly and there is no obvious of his responsibilities with regard to the
successor. compilation and maintenance of accounts
of the Central Government due to the
 Dismissal of the council of ministers
separation of accounts from audit, that is,
when it cannot prove the confidence
departmentalisation of accounts.
in the Lok Sabha.
 Statement 3 is incorrect: He audits all
 Dissolution of the Lok Sabha if the
expenditure from the Contingency Fund
council of ministers has lost its
of India and the Public Account of India
majority.
as well as the contingency fund of each
 Statement 3 is correct: After the 44th state and the public account of each
Constitutional Amendment Act of 1978 state.
the President may return a matter once  The duties and functions of the CAG as
for reconsideration of his ministers, but laid down by the Parliament and the
the reconsidered advice shall be binding. Constitution are:
 Statement 2 is correct: Pocket Veto: In  He audits the accounts related to all
this veto President neither ratifies nor expenditure from the Consolidated
rejects nor returns the bill, but simply keeps Fund of India, consolidated fund of
the bill pending for an indefinite period. each state and consolidated fund of
This power of the President not to take each union territory having a Legislative
any action (either positive or negative) on Assembly.

8
Google it:- PDF4Exams GS SCORE
https://t.me/pdf4exams «Downloaded From» https://t.me/testseries4exams

 He audits all expenditure from the  He ascertains and certifies the net
Contingency Fund of India and the proceeds of any tax or duty (Article
Public Account of India as well as the 279). His certificate is final. The ‘net
contingency fund of each state and proceeds’ means the proceeds of a tax
the public account of each state. or a duty minus the cost of collection.

 He audits all trading, manufacturing,  He acts as a guide, friend and


philosopher of the Public Accounts
profit and loss accounts, balance
Committee of the Parliament.
sheets and other subsidiary accounts
kept by any department of the Central  He compiles and maintains the
Government and state governments. accounts of state governments. In 1976,
he was relieved of his responsibilities
 He audits the receipts and expenditure with regard to the compilation and
of the Centre and each state to satisfy maintenance of accounts of the
him-self that the rules and procedures Central Government due to the
in that behalf are designed to secure separation of accounts from audit, that
an effective check on the assessment, is, departmentalisation of accounts.
collection and proper allocation of

RE
revenue. 18. Correct Option: (d)
 He audits the receipts and expenditure Explanation:
of the following:
Accountability checks on executive:
 All bodies and authorities
Accountability is enforced through the
substantially financed from the following mechanisms:
Central or state revenues;
 Government companies; and
 Other corporations and bodies,
O  Statement 1 is correct: Vote on Budget
by the Lok Sabha. After the presentation
of budget in Lok Sabha it must be passed
when so required by related laws. by the simple majority.
SC
 Statement 2 is correct: During Voting on
 He audits all transactions of the Central
demands for grants stage, the members
and state governments related to debt, of Parliament can discuss the details of
sinking funds, deposits, advances, the budget. They can also move motions
suspense accounts and remittance to reduce any demand for grant. Such
business. He also audits receipts, stock motions are called as ‘cut motion’, which
accounts and others, with approval of are of three kinds: Policy Cut, Economy
GS

the President, or when required by the Cut and Token Cut.


President.  Statement 3 is correct: Close scrutiny by
 He audits the accounts of any other Departmental Standing Committees: They
authority when requested by the are responsible for the administration and
President or Governor. For example, scrutiny of budgetary proposals and Bills
of Ministries/Departments.
the audit of local bodies.
 Statement 4 is correct: CAG audits
 He advises the President with regard
the accounts related to all expenditure
to prescription of the form in which the from the Consolidated Fund of India,
accounts of the Centre and the states consolidated fund of each state and
shall be kept (Article 150). consolidated fund of each union territory
 He submits his audit reports relating to having a Legislative Assembly.
the accounts of the Centre to President, Through Public Accounts Committee,
who shall, in turn, place them before Estimates Committee, Committee on
both the Houses of Parliament (Article Public Undertakings: These three Financial
151). Committees keep an un-remitting vigil over
Government spending and performance
 He submits his audit reports relating and bring to light inefficiencies, waste
to the accounts of a state to governor, and indiscretion in the implementation
who shall, in turn, place them before of programmes and policies approved by
the state legislature (Article 151). Parliament.

GS SCORE Google it:- PDF4Exams


9
https://t.me/pdf4exams «Downloaded From» https://t.me/testseries4exams

19. Correct Option: (d)  The substitute motion, being in the nature
of an amendment to the original motion,
Explanation: is also placed before the House and is
Amendment to the Constitution: discussed along with the original motion.
Those provisions of the Constitution which are  Statement 2 is correct: Amendments to
related to the federal structure of the polity substitute motions are not permissible.
can be amended by a special majority of the  At the end of the debate, the substitute
Parliament and also with the consent of half of motion only is put to the vote of the
the state legislatures by a simple majority. House.
The following provisions can be amended
in this way: 21. Correct Option: (b)
 Statement 3 is correct: Election of the Explanation:
President and its manner.
Equality of Power and Status between Lok
 Statement 2 is correct: Extent of the
Sabha & Rajya Sabha:
executive power of the Union and the
states.  Statement 1 is incorrect: Money Bill can

RE
be introduced only in the Lok Sabha.
 Supreme Court and high courts.
 Statement 4 is incorrect: A resolution
 Distribution of legislative powers between
for the discontinuance of the national
the Union and the states.
emergency can be passed only by the
 Any of the lists in the Seventh Schedule. Lok Sabha and not by the Rajya Sabha.
 Statement 1 is correct: Power of In the following matters, the powers and
O
Parliament to amend the Constitution and

status of the Rajya Sabha are equal to that
its procedure (Article 368 itself). of the Lok Sabha:
 Statement 4 is correct: Representation  Introduction and passage of ordinary
of states in Parliament. bills.
SC
 Introduction and passage of
20. Correct Option: (b) Constitutional amendment bills.
Explanation:  Introduction and passage of financial
bills involving expenditure from the
Substitute Motion:
Consolidated Fund of India.
 Statement 1 is incorrect: Motions moved
 Election and impeachment of the
GS

in substitution of the original motion


President.
for taking into consideration a policy
or situation or statement or any other  Election and removal of the Vice-
matter are called substitute motions. Such President. However, Rajya Sabha
motions, though drafted in such a way as alone can initiate the removal of the
to be capable of expressing an opinion vice-president. He is removed by a
by themselves are not, strictly speaking, resolution passed by the Rajya Sabha
substantive motions in as much as they by a special majority and agreed to by
depend upon the original motion. the Lok Sabha by a simple majority.
 After the original motion being be taken  Making recommendation to the
into consideration has been placed President for the removal of Chief
before the House and the mover of the Justice and judges of Supreme Court
motion has concluded the speech, the and High Courts, Chief Election
House proceeds to discuss the matter. Commissioner and Comptroller and
No further question is put thereon on Auditor General.
conclusion of the debate. However, before
 Statement 2 is correct: Approval of
the discussion commences, a member
ordinances issued by the President.
can move a substitute motion which,
while conforming to the subject matter of  Statement 3 is correct: Approval of
the original motion, is so drawn up as to proclamation of all three types of
express an opinion of the House. emergencies by the President.

10
Google it:- PDF4Exams GS SCORE
https://t.me/pdf4exams «Downloaded From» https://t.me/testseries4exams

 Selection of ministers including the 23. Correct Option: (b)


Prime Minister. Under the Constitution,
Explanation:
the ministers including the Prime
Minister can be members of either Council of Minister:
House. However, irrespective of their
 Statement 1 is incorrect: The ministers
membership, they are responsible only
of state can either be given independent
to the Lok Sabha.
charge of ministries/departments or
 Consideration of the reports of the can be attached to cabinet ministers.
constitutional bodies like Finance In case of attachment, they may either
Commission, Union Public Service be given the charge of departments of
Commission, comptroller and auditor the ministries headed by the cabinet
general, etc. ministers or allotted specific items of
work related to the ministries headed by
 Enlargement of the jurisdiction of the
cabinet ministers. In both the cases, they
Supreme Court and the Union Public work under the supervision and guidance
Service Commission. as well as under the overall charge and
responsibility of the cabinet ministers.

RE
22. Correct Option: (c)  Statement 2 is incorrect: Collective
Explanation: Responsibility means that all the
ministers own joint responsibility to the
Presidential Electoral College: Lok Sabha for all their acts of omission
 Statement 4 is incorrect: The voting is and commission. They work as a team
done by secret ballot. and swim or sink together. When the


O
Statement 1 is correct: The President is
elected by an Electoral College consisting
Lok Sabha passes a no-confidence motion
against the council of ministers, all the
ministers have to resign including those
of the elected members of both Houses ministers who are from the Rajya Sabha.
SC
of Parliament i.e. Lok Sabha and Rajya Alternatively, the council of ministers can
Sabha and of the State Legislative advise the president to dissolve the Lok
Assemblies (Vidhan Sabhas). Sabha on the ground that the House does
 Statement 2 is incorrect: Thus, the not represent the views of the electorate
nominated members of both of Houses faithfully and call for fresh elections. The
of Parliament, the nominated members President may not oblige the council of
of the state legislative assemblies, the ministers that has lost the confidence of
GS

members (both elected and nominated) of the Lok Sabha.


the state legislative councils (in case of the  Statement 3 is correct: Council of
bicameral legislature) and the nominated minister is headed by Prime Minister.
members of the Legislative Assemblies of
Delhi and Puducherry do not participate
24. Correct Option: (a)
in the election of the President.
Explanation:
 Where an assembly is dissolved, the
members cease to be qualified to vote Qualifications for Election of President:
in Presidential election, even if fresh A person to be eligible for election as President
elections to the dissolved assembly are should fulfil the following qualifications:
not held before the Presidential election.
 He should be a citizen of India.
 The Constitution provides that there shall
be uniformity in the scale of representation  Statement 3 is incorrect: He should have
of different states as well as parity between completed 35 years of age.
the states as a whole and the Union at the  Statement 1 is incorrect: He should be
election of the President. qualified for election as a member of the
 Statement 3 is correct: The election is Lok Sabha.
held by means of single transferable vote  Statement 2 is correct: He should not
system of proportional representation. hold any office of profit under the Union

GS SCORE Google it:- PDF4Exams


11
https://t.me/pdf4exams «Downloaded From» https://t.me/testseries4exams

government or any state government or 26. Correct Option: (c)


any local authority or any other public
Explanation:
authority. A sitting President or Vice-
President of the Union, the Governor of Comptroller and Auditor General of India
any state and a minister of the Union (CAG):
or any state is not deemed to hold any
 Statement 2 is incorrect: No minister
office of profit and hence qualified as a
can represent the CAG in Parliament
presidential candidate.
(both Houses) and no minister can be
called upon to take any responsibility for
25. Correct Option: (d) any actions done by him.
Explanation:  The Constitution of India (Article 148)
provides for an independent office of the
Parliament of India: Comptroller and Auditor General of India
 Statement 1 is incorrect: Lok Sabha (CAG). He is the head of the Indian Audit
represents People of India as a whole. and Accounts Department.
 Statement 1 is correct: CAG is the
 Statement 2 is incorrect: Rajya Sabha
guardian of the public purse and controls

RE
represents States and the Union
the entire financial system of the
Territories. country at both the levels, the Centre
 Statement 3 is incorrect: The President and the state. His duty is to uphold the
of India is not a member of either House Constitution of India and laws of Parliament
of Parliament and does not sit in the in the field of financial administration.
Parliament to attend its meetings,  Statement 3 is correct: He advises the
but he is an integral part of the
O President about prescription of the form
Parliament. in which the accounts of the Centre and
the states shall be kept (Article 150).
 Under the Constitution, the Parliament
President prescribes format of accounts
of India consists of three parts viz, the
SC
to keep.
President, the Council of States and the
House of the People. In 1954, the Hindi  Some of the duties and functions of the
names ‘Rajya Sabha’ and ‘Lok Sabha’ were CAG as laid down by the Parliament and
adopted by the Council of States and the the Constitution are:
House of People respectively.  He audits the accounts related to all
expenditure from the Consolidated
 The Rajya Sabha is the Upper House
Fund of India, Consolidated Fund of
GS

(Second Chamber or House of Elders) each state and Consolidated fund of


which represents the states and union each union territory having a Legislative
territories of the Indian Union and the Lok Assembly.
Sabha is the Lower House (First Chamber
 He audits all expenditure from the
or Popular House) which represents the
Contingency Fund of India and the
people of India as a whole.
Public Account of India as well as the
 Though the President of India is not a Contingency fund of each state and
member of either House of Parliament and the public account of each state.
does not sit in the Parliament to attend  He audits all trading, manufacturing,
its meetings, he is an integral part of the profit and loss accounts, balance
Parliament. This is because a bill passed sheets and other subsidiary accounts
by both the Houses of Parliament cannot kept by any department of the Central
become law without the President’s assent. Government and state governments.
He also performs certain functions relating  He audits the receipts and expenditure
to the proceedings of the Parliament, for of the Centre and each state to satisfy
example, he summons and pro-rogues himself that the rules and procedures
both the Houses, dissolves the Lok in that behalf are designed to secure
Sabha, addresses both the Houses, issues an effective check on the assessment,
ordinances when they are not in session, collection and proper allocation of
and so on. revenue.

12
Google it:- PDF4Exams GS SCORE
https://t.me/pdf4exams «Downloaded From» https://t.me/testseries4exams

27. Correct Option: (a)  He must make and subscribe to an


oath or affirmation before the person
Explanation:
authorised by the Election Commission
State legislative council: for this purpose. In his oath or
affirmation, he swears: To bear true
 Statement 1 is incorrect: Legislative
faith and allegiance to the Constitution
assembly of State is appointed for five
of India and to uphold the sovereignty
years and not the legislative council.
and integrity of India.
Like the Rajya Sabha, the legislative
council is a continuing chamber, that is,  Statement 3 is correct: He must be
it is a permanent body and is not subject not less than 30 years of age in the
to dissolution. case of the legislative council and not
less than 25 years of age in the case
 Statement 2 is incorrect: According of the legislative assembly.
to Article 171 (1) the total number of
members in the Legislative Council of
a State shall not exceed one-third of 28. Correct Option: (c)
the total number of members in the Explanation:

RE
Legislative Assembly of that State and
Governor’s office:
not the fifty percent.
 Statement 1 is incorrect: The Constitution
 The members of the legislative council
does not lay down any grounds upon
are indirectly elected. The maximum
which a Governor may be removed by
strength of the council is fixed at one-
the President.
third of the total strength of the assembly
and the minimum strength is fixed at
O
40. It means that the size of the council
depends on the size of the assembly of
 A governor holds office for a term of five
years from the date on which he enters
upon his office. This term of five years is
subject to the pleasure of the President.
SC
the concerned state. This is done to ensure
the predominance of the directly elected  He can resign at any time by addressing
House (assembly) in the legislative affairs a resignation letter to the President. The
of the state. governor has no security of tenure and no
 Though the Constitution has fixed the fixed term of office. He may be removed
maximum and the minimum limits, the by the President at any time.
actual strength of a Council is fixed by The Constitution does not lay down any
GS


Parliament. grounds upon which a governor may be
 Like the Rajya Sabha, the legislative removed by the President. The President
council is a continuing chamber, that is, it may transfer a Governor appointed to one
is a permanent body and is not subject to state to another state for the rest of the
dissolution. But, one-third of its members term. Further, a Governor whose term has
retire on the expiration of every second expired may be reappointed in the same
year. So, a member continues as such for state or any other state.
six years.  Statement 2 is correct: A governor can
hold office beyond his term of five
 The vacant seats are filled up by fresh
years until his successor assumes charge.
elections and nominations (by governor)
The underlying idea is that there must be
at the beginning of every third year. The
governor in the state and there cannot be
retiring members are also eligible for re-
an interregnum.
election and re-nomination any number
of times.  Statement 3 is correct: A governor
possesses executive, legislative, financial
 The Constitution lays down the following
and judicial power more or less analogous
qualifications for a person to be chosen a
to the President of India. However, he has
member of the state legislature:
no diplomatic, military or emergency
 He must be a citizen of India. powers like the President.

GS SCORE Google it:- PDF4Exams


13
https://t.me/pdf4exams «Downloaded From» https://t.me/testseries4exams

29. Correct Option: (d)  The All India Judicial Service referred
to in clause (1) shall not include any
Explanation:
post inferior to that of a district judge
Prime Minister of India: as defined in article 236.
 Statement 1 is incorrect: Article 74  The law providing for the creation of
(2) says that: There shall be a Council the all India judicial service aforesaid
of Ministers with the Prime Minister at may contain such provisions for the
the head to aid and advise the President amendment of Chapter VI of Part VI
who shall, in the exercise of his functions, as may be necessary for giving effect
act in accordance with such advice. to the provisions of that law and no
(Provided that the President may require
such law shall be deemed to be an
the Council of Ministers to reconsider
amendment of this Constitution for
such advice, either generally or otherwise,
and the President shall act in accordance the purposes of article 368
with the advice tendered after such  Advantages of AIJS:
reconsideration).
 Statement 2 is correct: Creating
 Statement 2 is incorrect: President an All India Judicial Service would

RE
must act in accordance with the advice make judiciary more accountable,
tendered by the Prime Minister but not professional and equitable.
always. Though the President has no
constitutional discretion, he has some  Given the strength of the judiciary in
situational discretion. In other words, the subordinate courts is over a fifth short
President can act on his discretion (that is, of the total number of the sanctioned
without the advice of the ministers) under posts, such a move is likely to help
the following situations:
O ease pendency.
 Appointment of Prime Minister when  Only the judges of proven competence
no party has a clear majority in the Lok will preside over the benches and it
Sabha or when the Prime Minister in
SC
will minimize the scope of aberraƟon,
office dies suddenly and there is no
arbitrariness and nepoƟsm in judiciary.
obvious successor.
 Dismissal of the council of ministers
when it cannot prove the confidence 31. Correct Option: (d)
of the Lok Sabha. Explanation:
 Dissolution of the Lok Sabha if the Upper House of Indian Parliament
GS

council of ministers has lost its


majority.  Statement 2 is correct: Upper house, i.e.
Rajya Sabha has 245 members. Of these,
229 members represent the states, 4
30. Correct Option: (d)
members represent the union territories
Explanation: and 12 members are nominated by the
All India Judicial Services President.

 The proposal for an All-India Judicial  The Fourth Schedule of the Constitution
Service (AIJS) in lines of All-India Services deals with the allocation of seats in the
was proposed as early as 1950. The Rajya Sabha to the states and union
idea was first mooted by the Law territories.
Commission in the 1950s to have an  Statement 3 is correct: The representatives
All-India Judicial Services.
of states in the Rajya Sabha are elected
 Statement 1 is correct: The Constitution by the elected members of state
of India was amended in 1977 to provide legislative assemblies. The election is
for an All-India Judicial Services under held in accordance with the system of
Article 312 on recommendation of proportional representation by means
Swaran Singh Committee. of the single transferable vote. The seats
 Article 312 in The Constitution Of India are allotted to the states in the Rajya
1949 Sabha on the basis of population.

14
Google it:- PDF4Exams GS SCORE
https://t.me/pdf4exams «Downloaded From» https://t.me/testseries4exams

 The representatives of each union territory qualified to be appointed a judge of the


in the Rajya Sabha are indirectly elected by Supreme Court. In other words, he must
members of an electoral college specially be a citizen of India and he must have
constituted for the purpose. been a judge of some high court for
five years or an advocate of some high
 Statement 1 is correct: Out of the seven
court for ten years or an eminent jurist,
union territories, only two (Delhi and
in the opinion of the President.
Puducherry) have representation in
Rajya Sabha. The populations of other  The term of office of the AG is not fixed
five union territories are too small to have by the Constitution. The Constitution
any representative in the Rajya Sabha. does not contain the procedure and
grounds for his removal. He holds office
 Nominated Members: The president
during the pleasure of the president. This
nominates 12 members to the Rajya
means that he may be removed by the
Sabha from people who have special
president at any time. He may also quit
knowledge or practical experience in art,
his office by submitting his resignation
literature, science and social service.
to the president. Conventionally, he
resigns when the government (council of

RE
32. Correct Option: (d) ministers) resigns or is replaced, as he is
appointed on its advice.
Explanation:
 Statement 1 is incorrect: Attorney general
Functioning of the Indian Parliament: is not a member of Parliament. He
 Statement 1 and 2 are incorrect: resigns when the government (council
The Constitution has declared Hindi of ministers) resigns or is replaced, as
he is appointed on its advice.
and English to be the languages for

However, the presiding officer can


permit a member to address the House
O
transacting business in the Parliament.  As the chief law officer of the
Government of India, the duties of the
AG include the following:
in his mother-tongue.
SC
 To give advice to the Government of
 In both the Houses, arrangements are India upon such legal matters, which
made for simultaneous translation. are referred to him by the president.
Though English was to be discontinued
as a floor language after the expiration  To perform such other duties of a legal
of fifteen years from the commencement character that are assigned to him by
of the Constitution (that is, in 1965), the the president.
GS

Official Languages Act (1963) allowed  To discharge the functions conferred


English to be continued along with on him by the Constitution or any
Hindi. other law.
 Statement 3 is correct: In addition to  Statement 3 is incorrect: In accordance
the members of a House, every minister with Article 76, he must perform duties
and the attorney general of India have assigned to him by the President.
the right to speak and take part in the
proceedings of either House, any joint
sitting of both the Houses and any
34. Correct Option: (c)
committee of Parliament of which he Explanation:
is a member, without being entitled to
vote. Lok Sabha:
 Statement 2 is incorrect: The maximum
33. Correct Option: (d) strength of the Lok Sabha is fixed at
552. Out of this, 530 members are to
Explanation: be the representatives of the states, 20
members are to be the representatives of
Attorney General of India:
the union territories and 2 members are
 Statement 2 is incorrect: The Attorney to be nominated by the president from
General (AG) is appointed by the the Anglo-Indian community. At present,
President. He must be a person who is the Lok Sabha has 545 members.

GS SCORE Google it:- PDF4Exams


15
https://t.me/pdf4exams «Downloaded From» https://t.me/testseries4exams

 The representatives of states in the Lok the functions of the President, he does
Sabha are directly elected by the people not perform the duties of the office of the
from the territorial constituencies in Chairman of Rajya Sabha.
the states. The election is based on the
principle of universal adult franchise.
36. Correct Option: (a)
 Statement 1 is incorrect: Every Indian
citizen who is above 18 years who has Explanation:
been listed in electoral list and who is Council of Ministers:
not disqualified under the provisions of
the Constitution or any law is eligible  Option (a) is correct: The Prime Minister
to vote at election. is appointed by the President, while the
other ministers are appointed by the
 Statement 3 is correct: The voting age President on the advice of the Prime
was reduced from 21 to 18 years by the Minister. This means that the President
61st Constitutional Amendment Act, can appoint only those persons as
1988. ministers who are recommended by the
 The Parliament has enacted the Union Prime minister.

RE
Territories (Direct Election to the House
 A minister who is a member of one House
of the People) Act, 1965, by which the
of Parliament has the right to speak and
members of Lok Sabha from the union
to take part in the proceedings of the
territories are also chosen by direct
other House also, but he can vote only in
election.
the House of which he is a member.
Option (b) is incorrect: In accordance
35. Correct Option: (b) 
O with the conventions of the parliamentary
Explanation: system of government, the President
must appoint the leader of the majority
Leader of the House:
party in the Lok Sabha as the Prime
SC
 Statement 1 is incorrect: Under the Minister.
Rules of Lok Sabha, the ‘Leader of the
House’ means the Prime Minister, if  Article 75. (1) The Prime Minister shall
he is a member of the Lok Sabha, or a be appointed by the President and the
minister who is a member of the Lok other Ministers shall be appointed by
Sabha and is nominated by the Prime the President on the advice of the Prime
Minister (Cabinet).
Minister to function as the Leader of
GS

the House.  Option (c) is incorrect: Prime minister


chooses his kitchen cabinet, and not
 There is also a ‘Leader of the House’ in the
the President. The cabinet, a small body
Rajya Sabha. He is a minister and a member
consisting of the Prime Minister as its
of the Rajya Sabha and is nominated by
head and some 15 to 20 most important
the prime minister to function as such.
ministers, is the highest decision-making
 The leader of the house in either House body in the formal sense. However, a still
is an important functionary and exercises smaller body called the ‘Inner Cabinet’
direct influence on the conduct of business. or ‘Kitchen Cabinet’ has become the
He can also nominate a deputy leader of real centre of power. This informal body
the House. The same functionary in USA consists of the Prime Minister and two to
is known as the ‘majority leader’. four influential colleagues in whom he has
faith and with whom he can discuss every
Chairman of Rajya Sabha:
problem. It advises the prime minister
 Statement 2 is correct: The presiding on important political and administrative
officer of the Rajya Sabha is known as issues and assists him in making crucial
the Chairman. The Vice-President of decisions. It is composed of not only
India is the ex-officio Chairman of the cabinet ministers but also outsiders like
Rajya Sabha. friends and family members of the Prime
Minister.
 During any period when the Vice-
President acts as President or discharges  Option (d) is incorrect: Prime minister

16
Google it:- PDF4Exams GS SCORE
https://t.me/pdf4exams «Downloaded From» https://t.me/testseries4exams

can choose any body as minister but life of the existing House, and a new
in 6 months he/she must become House is constituted after general
member of either house of Parliament. elections are held.
The members of Parliament, either Lok
Sabha or Rajya Sabha, are appointed as 38. Correct Option: (d)
ministers. A person who is not a member
of either House of Parliament can also Explanation:
be appointed as a minister. But, within
President Election:
six months, he must become a member
(either by election or by nomination) of  Statement 1 is incorrect: All doubts and
either House of Parliament, otherwise, he disputes in connection with election
ceases to be a minister. of the President are inquired into and
decided by the Supreme Court whose
decision is final. The election of a person
37. Correct Option: (b)
as President cannot be challenged on
Explanation: the ground that the Electoral College
was incomplete (i.e. existence of any
Parliament Sessions:
vacancy among the members of Electoral

RE
 Statement 2 is incorrect: Adjournment: College).
A session of Parliament consists of many
 Qualifications for Election as President:
meetings. Each meeting of a day consists
of two sittings, that is, a morning sitting A person to be eligible for election as
from 11 am to 1 pm and post-lunch sitting President should fulfil the following
from 2 pm to 6 pm. A sitting of Parliament
O qualifications:
can be terminated by adjournment or  Statement 3 is incorrect: He should be
adjournment sine die or prorogation or a citizen of India. No such amendment
dissolution (in the case of the Lok Sabha). has taken place in 2016.
An adjournment suspends the work in a
 He should have completed 35 years
SC
sitting for a specified time, which may
of age.
be hours, days or weeks.
 Adjournment Sine Die: It means  He should be qualified for election
terminating a sitting of Parliament as a member of the Lok Sabha.
for indefinite period. In other words,  He should not hold any office of profit
when the House is adjourned without under the Union government or any
naming a day for reassembly, it is called state government or any local authority
GS

adjournment sine die. The power of or any other public authority.


adjournment as well as adjournment sine
die lies with the presiding officer of the  Statement 2 is incorrect: Term of
House. He can also call a sitting of the President’s Office: The President holds
House before the date or time to which office for a term of five years from the
it has been adjourned or at any time after date on which he enters upon his office.
the House has been adjourned sine die. However, he can resign from his office at
any time by addressing the resignation
 Statement 1 is incorrect: Prorogation:
letter to the Vice-President. Further, he
The presiding officer (Speaker or
can also be removed from the office
Chairman) declares the House adjourned
sine die, when the business of a session before completion of his term by the
is completed. Within the next few days, process of impeachment. However he
the President issues a notification for can be reappointed any number of
prorogation of the session. However, the times.
President can also prorogue the House  The President can hold office beyond
while in session. his term of five years until his successor
 Statement 3 is correct: Dissolution: assumes charge. He is also eligible for re-
Rajya Sabha, being a permanent House, election to that office. He may be elected
is not subject to dissolution. Only the Lok for any number of terms. However, in USA,
Sabha is subject to dissolution. Unlike a a person cannot be elected to the office
prorogation, dissolution ends the very of the President more than twice.

GS SCORE Google it:- PDF4Exams


17
https://t.me/pdf4exams «Downloaded From» https://t.me/testseries4exams

39. Correct Option: (a) Explanation:


Explanation: Territorial Extent of Central and State
Legislation:
Bills in Parliament:
 Statement 2 is incorrect: Parliament
Ordinary Bills: can made law for whole or any part of
 Ordinary bills are concerned with any country not economic territory.
matter other than Financial Bills, money  The Constitution defines the territorial
Bills and Constitutional Amendment limits of the legislative powers vested in
Bills. the Centre and the states in the following
way:
 Such Bills can be introduced in either
House of Indian Parliament (Lok  The Parliament can make laws for the
Sabha or Rajya Sabha) without the whole or any part of the territory of
recommendations of President of India. India. The territory of India includes the
These bills are passed by Simple Majority states, the union territories, and any
in both Houses. other area for the time being included
in the territory of India.

RE
 Statement 1 is correct: If Ordinary Bill is
defeated in the Lok Sabha; it may lead to  A state legislature can make laws for
resignation of government if introduced the whole or any part of the state.
by a Minister. The laws made by a state legislature
are not applicable outside the state,
Money Bills: except when there is a sufficient nexus
between the state and the object.
These are defined in Article 110. These

O
Bills deal with the taxes, borrowings,  The Parliament alone can make ‘extra-
consolidated and contingency funds, territorial legislation’. Thus, the laws of
audit and accounting, etc. the Parliament are also applicable to
the Indian citizens and their property
SC
 Article 109 of Indian Constitution gives in any part of the world.
special procedure regarding Money Bills.
Distribution of Legislative Subjects:
 A money Bill can originate only in Lok
Sabha after the recommendations of the  The Constitution provides for a three-
President. Through the Bill is sent to Rajya fold distribution of legislative subjects
Sabha also but even Rajya Sabha rejects/ between the Centre and the states, viz.,
returns the Bill (within days necessarily), List-I (the Union List), List-II (the State List)
GS

the Bill is deemed to be passed. and List-III (the Concurrent List) in the
Seventh Schedule.
 The Appropriation Bill and Annual
 The Parliament has exclusive powers
Financial Bill (Budget) are Money Bills.
to make laws with respect to any of the
 Money bill can only be introduced in Lok matters enumerated in the Union List.
Sabha by Minister. Rajya Sabha has limited
 The state legislature has “in normal
powers with respect to the Money Bills. It
circumstances” exclusive powers to make
cannot reject or amend the bill, it can only laws with respect to any of the matters
recommend the amendments. A money enumerated in the State List.
bill must be returned to the Lok Sabha
within 14 days, or the bill is deemed to  Statement 1 is correct: Both, the
have passed in both houses in the form Parliament and state legislature can
it was originally passed by the Lok Sabha. make laws with respect to any of the
matters enumerated in the Concurrent
Rajya Sabha can only suggest changes
List.
without right to vote.
 The power to make laws with respect to
 Statement 2 is incorrect: If Money Bill
residuary subjects (i.e., the matters which
is defeated in Lok Sabha, Government
are not enumerated in any of the three
must resign with immediate effect. lists) is vested in the Parliament. This
residuary power of legislation includes
40. Correct Option: (a) the power to levy residuary taxes.

18
Google it:- PDF4Exams GS SCORE
https://t.me/pdf4exams «Downloaded From» https://t.me/testseries4exams

41. Correct Option: (c) 42. Correct Option: (b)


Explanation: Explanation:
Impeachment of President: Writ jurisdiction of the Supreme Court and
High Court:
 Statement 2 is incorrect: Article 61 (1)
When a President is to be impeached for The writ jurisdiction of the Supreme Court
violation of the Constitution, the charge differs from that of a high court in three
shall be preferred by either House of respects:
Parliament. Nowhere in the constitution,  Statement 1 is incorrect: The Supreme
had the violation of constitution Court can issue writs only for the
defined. enforcement of fundamental rights
whereas a high court can issue writs
 Statement 1 is correct: The President
not only for the enforcement of
can be removed from office by a process
Fundamental Rights but also for any
of impeachment for ‘violation of the
other purpose. The expression ‘for any
Constitution’. However, the Constitution other purpose’ refers to the enforcement
does not define the meaning of the phrase of an ordinary legal right. Thus, the writ

RE
‘violation of the Constitution’. jurisdiction of the Supreme Court, in this
 Statement 3 is incorrect: The respect, is narrower than that of high
impeachment charges can be initiated court.
by either House of Parliament. These  Statement 2 is incorrect: The Supreme
charges should be signed by one-fourth Court can issue writs against a person or
members of the House (that framed the government throughout the territory
charges), and a 14 days’ notice should be of India whereas a high court can issue


given to the President.
After the impeachment resolution is
O writs against a person residing or against
a government or authority located
within its territorial jurisdiction only
passed by a majority of two-thirds of
or outside its territorial jurisdiction
SC
the total membership of that House, it
only if the cause of action arises within
is sent to the other House, which should
its territorial jurisdiction. Thus, the
investigate the charges.
territorial jurisdiction of the Supreme
 The President has the right to appear and Court for the purpose of issuing writs is
to be represented at such investigation. If wider than that of a high court.
the other House also sustains the charges  Statement 3 is correct: A remedy under
and passes the impeachment resolution Article 32 is in itself a Fundamental Right
GS

by a majority of two-thirds of the total and hence, the Supreme Court may not
membership, then the President stands refuse to exercise its writ jurisdiction. On
removed from his office from the date on the other hand, a remedy under Article
which the resolution is so passed. 226 is discretionary and hence, a high
court may refuse to exercise its writ
 Thus, an impeachment is a quasi-judicial
jurisdiction. Article 32 does not merely
procedure in the Parliament. In this confer power on the Supreme Court as
context, two things should be noted: Article 226 does on a high court to issue
 The nominated members of either writs for the enforcement of fundamental
House of Parliament can participate rights or other rights as part of its general
in the impeachment of the President jurisdiction. The Supreme Court is thus
though they do not participate in his constituted as a defender and guarantor
of the fundamental rights.
election;
 The elected members of the legislative
43. Correct Option: (b)
assemblies of states and the Union
Territories of Delhi and Puducherry do Explanation:
not participate in the impeachment of Functions of the Prime Minister:
the President though they participate
in his election.  Option (a) is incorrect: Article 75(6): The
salaries and allowances of Ministers
 No President has so far been impeached. shall be such as Parliament may from

GS SCORE Google it:- PDF4Exams


19
https://t.me/pdf4exams «Downloaded From» https://t.me/testseries4exams

time to time by law determine and, becomes competent to make laws on that
until Parliament so determines, shall be as matter.
specified in the Second Schedule.
 During a National Emergency: The
 Option (c) is incorrect: The Speaker is Parliament acquires the power to legislate
elected by the Lok Sabha from amongst with respect to matters in the State
its members; President has no role to List, while a proclamation of national
play. emergency is in operation.
 Option (d) is incorrect: Funds are  When States Make a Request: When the
granted by Parliament to each ministry,
legislatures of two or more states pass
Prime Minster do not play major role in
resolutions requesting the Parliament to
this context.
enact laws on a matter in the State List,
 The Prime Minister enjoys the following then the Parliament can make laws for
powers as head of the Union Council of regulating that matter. A law so enacted
Ministers: applies only to those states which have
 He recommends persons who can passed the resolutions. However, any
be appointed as ministers by the other state may adopt it afterwards by
President. The President can appoint passing a resolution to that effect in its

RE
only those persons as ministers who legislature. Such a law can be amended
are recommended by the Prime or repealed only by the Parliament and
Minister. not by the legislatures of the concerned
 He allocates and reshuffles various states.
portfolios among the ministers.
 Option (b) is correct: To Implement
 Option (b) is correct: He can ask International Agreements: The
a minister to resign or advise the
O Parliament can make laws on any matter
President to dismiss him in case of in the State List for implementing the
difference of opinion. international treaties, agreements or
 He presides over the meeting of conventions. This provision enables the
SC
council of ministers and influences its Central government to fulfil its international
decisions. obligations and commitments.
 He guides, directs, controls, and  During President’s Rule: When the
coordinates the activities of all the President’s rule is imposed in a state, the
ministers. Parliament becomes empowered to make
 He can bring about the collapse of the council laws with respect to any matter in the
State List in relation to that state.
GS

of ministers by resign in from office.

44. Correct Option: (b) 45. Correct Option: (a)


Explanation: Explanation:
Legislative powers of the Parliament: Chief Minister in State:
 The distribution of legislative powers  Statement 1 is incorrect: In the scheme
between the Centre and the states is of Parliamentary system of government
to be maintained in normal times. But, provided by the Constitution, the governor
in abnormal times, the distribution is is the nominal executive authority (de
either modified or suspended. In other jure executive) and the Chief Minister is
words, the Constitution empowers the
the real executive authority (de facto
Parliament to make laws on any matter
executive). In other words, the governor
enumerated in the State List Without
is the head of the state while the Chief
consent of any State under the following
five extraordinary circumstances: Minister is the head of the government.
Thus, the position of the Chief Minister at
 When Rajya Sabha Passes a Resolution: the state level is analogous to the position
If the Rajya Sabha declares that it is of prime minister at the Centre.
necessary in the national interest that
Parliament should make laws on a matter  Statement 3 is incorrect: Before the Chief
in the State List, then the Parliament Minister enters his office, the Governor

20
Google it:- PDF4Exams GS SCORE
https://t.me/pdf4exams «Downloaded From» https://t.me/testseries4exams

administers to him the oaths of office  The President is elected not directly by
and secrecy. In his oath of secrecy, the the people but by members of electoral
Chief Minister swears that he will not college consisting of:
directly or indirectly communicate or  The elected members of both the
reveal to any person(s) any matter that Houses of Parliament.
is brought under his consideration or
 The elected members of the legislative
becomes known to him as a state minister
assemblies of the states.
except as may be required for the due
discharge of his duties as such minister.  Statement 3 is correct: The elected
members of the legislative assemblies
 Statement 2 is correct: The Constitution of the Union Territories of Delhi and
does not contain any specific procedure Puducherry.
for the selection and appointment of
 Thus, the nominated members of both
the Chief Minister. Article 164 only says
of Houses of Parliament, the nominated
that the Chief Minister shall be appointed members of the state legislative
by the governor. assemblies, the members (both elected
However, this does not imply that the and nominated) of the state legislative

RE

governor is free to appoint any one as councils (in case of the bicameral
the Chief Minister. In accordance with the legislature) and the nominated members
of the Legislative Assemblies of Delhi
convections of the Parliamentary system of
and Puducherry do not participate in the
government, the governor has to appoint
election of the President.
the leader of the majority party in the state
legislative assembly as the Chief Minister.

in the assembly, then the governor may


O
But, when no party has a clear majority

exercise his personal discretion in the


47. Correct Option: (c)
Explanation:

selection and appointment of the Chief Appointment of District Judges


SC
Minister.  Option ( c) is correct: The appointment,
posting and promotion of district judges
 In such a situation, the governor usually
in a state are made by the governor of
appoints the leader of the largest party the state in consultation with the Cheif
or coalition in the assembly as the Chief Justice of the high court.
Minister and ask him to seek a vote of
confidence in the House within a month.  A person to be appointed as district
GS

judge should have the following


qualifications:
46. Correct Option: (a)
 He should not already be in the
Explanation: service of the Central or the state
President of India: government.

 Statement 1 is incorrect: Union executive  He should have been an advocate or


consist of President, Vice-president, a pleader for seven years at bar.
Prime minister and council of Ministers  He should be recommended by
and Attorney General of India. the Chief Justice of high court for
 Statement 2 is incorrect: Qualification appointment.
for election of President is given in  District judges are also appointed by
Article 58 and Qualification for election way of elevation of judges from courts
of vice-President is given in Article 66. subordinate to district courts provided
 Articles 52 to 78 in Part V of the Constitution they fulfill the minimum years of
deal with the Union executive. service.
 The President is the head of the Indian  The district judge is the highest judicial
State. He is the first citizen of India and authority in the district. He possesses
acts as the symbol of unity, integrity and original and appellate jurisdiction in
solidarity of the nation. both civil as well as criminal matters.

GS SCORE Google it:- PDF4Exams


21
https://t.me/pdf4exams «Downloaded From» https://t.me/testseries4exams

48. Correct Option: (d) all cases in the Supreme Court in which
the Government of India is concerned.
Explanation:
 To represent the Government of India
Motion
in any reference made by the president
 MPs may raise issues of public to the Supreme Court under Article
importance in Parliament, and examine 143 of the Constitution.
the government’s response to problems
being faced by citizens through:  To appear (when required by the
Government of India) in any high court
 a debate, which entails a reply by the in any case in which the Government
concerned minister, or
of India is concerned.
 a motion which entails a vote.
 Following limitations are placed on the
 Using these methods, MPs may discuss Attorney General in order to avoid any
important matters, policies, and topical complication and conflict of duty:
issues.
 He should not advise or hold a brief
 Statement 1 is correct: Discussing against the Government of India.

RE
important issues (such as inflation,
drought, and corruption),  He should not advise or hold a brief
in cases in which he is called upon to
 Statement 2 is correct: Adjournment
advise or appear for the Government
of business in a House in order to
of India.
express displeasure over a government
policy,  Statement 2 is correct: He should not
O
 Statement 3 is correct: Expressing no defend accused persons in criminal
confidence in the government leading prosecutions without the permission
to its resignation. of the Government of India.
 He should not accept appointment
SC
49. Correct Option: (a) as a director in any company or
corporation without the permission
Explanation:
of the Government of India. However,
Attorney General of India: the Attorney General is not a full-time
 Statement 1 is incorrect: The Attorney counsel for the Government. He does
General is not a full-time counsel for not fall in the category of government
the Government. He does not fall in the servants. Further, he is not debarred
GS

category of government servants. from private legal practice.


 As the chief law officer of the
Government of India, the duties of the 50. Correct Option: (a)
AG include the following: Explanation:
 To give advice to the Government of
Lame-duck session:
India upon such legal matters, which
are referred to him by the president.  Statement 1 is correct: It is conducted
after election of new members but
 To perform such other duties of a legal
before they are installed.
character that are assigned to him by
the president.  Statement 2 is correct: It refers to the
last session of the existing Lok Sabha,
 To discharge the functions conferred
after a new Lok Sabha has been elected.
on him by the Constitution or any
Those members of the existing Lok Sabha
other law.
who could not get re-elected to the new
 The President has assigned the following Lok Sabha are called lame-ducks.
duties to the Attorney General of
 Lame-duck session do not usually occur
India:
in countries under a parliamentary form
 Statement 3 is correct: To appear on of government, whether the Westminster
behalf of the Government of India in system or the other modules.

22
Google it:- PDF4Exams GS SCORE
https://t.me/pdf4exams «Downloaded From» https://t.me/testseries4exams

 Statement 3 is incorrect: Lame-duck 52. Correct Option: (b)


session does not require minimum
number of members to transact the Explanation:
business of the House. Original jurisdiction of the Supreme
Court:
51. Correct Option: (a)  With regard to the exclusive original
Explanation: jurisdiction of the Supreme Court, it is
mandatory
Pardoning Power of the President:
 the dispute must involve a question
 The President shall have the power to (whether of law or fact) on which the
grant pardons, reprieves, respites or existence or extent of a legal right
remission of punishment or to suspend depends. Thus, the questions of
remit or commute the sentence of any political nature are excluded from it.
persons convicted of any offence.
 any suit brought before the Supreme
 In all cases where the punishment or Court by a private citizen against the
sentence is by a court martial. Centre or a state cannot be entertained

RE
 In all cases where the punishment or under this.
sentence is for an offence against any  Original jurisdiction of the Supreme Court
law relating to a matter to which the does not extend to the following:
executive power of the Union extends.
 A dispute arising out of any pre-
 In all cases where the sentence is a Constitution treaty, agreement,
sentence of death. O covenant, engagement, or other similar
 The pardoning power of the President instrument.
includes the following:  A dispute arising out of any treaty,
 Statement 1 is correct: Pardon: It agreement, etc., which specifically
provides that the said jurisdiction does
SC
removes both the sentence and the
conviction and completely absolves not extent to such a dispute.
the convict from all sentences,  Statement 2 is correct: Inter-state
punishments and disqualifications. water disputes.
 Statement 2 is incorrect:  Matters referred to the Finance
Commutation: It denotes the Commission.
substitution of one form of punishment
 Statement 4 is correct: Adjustment
GS

with a lighter form and also applicable


in case of court martial. of certain expenses and pensions
between the Centre and the states.
 Remission: It implies reducing the
period of sentence without changing  Statement 5 is correct: Ordinary
its character. For example, a sentence dispute of Commercial nature between
of rigorous imprisonment for two the Centre and the states.
years may be remitted to rigorous  Recovery of damages by a state against
imprisonment for one year.
the Centre.
 Statement 3 is incorrect: Respite: It
 Statement 1 is incorrect: In federal
denotes awarding a lesser sentence in
disputes, the Supreme Court has exclusive
place of one originally awarded due to
some special fact, such as the physical original jurisdiction. Exclusive means, no
disability of a convict or the pregnancy other court can decide such disputes and
of a woman offender. original means, the power to hear such
disputes in the first instance, not by way
 Statement 4 is incorrect: Reprieve: of appeal.
It implies a stay of the execution of
a sentence (especially that of death)  Statement 3 is incorrect: The Supreme
for a temporary period. Its purpose is Court is empowered to issue writs
to enable the convict to have time to including habeas corpus, mandamus,
seek pardon or commutation from the prohibition, quo-Warranto and certiorari
President. for the enforcement of the fundamental

GS SCORE Google it:- PDF4Exams


23
https://t.me/pdf4exams «Downloaded From» https://t.me/testseries4exams

rights of an aggrieved citizen. In this 54. Correct Option: (d)


regard, the Supreme Court has original
Explanation:
jurisdiction in the sense that an aggrieved
citizen can directly go to the Supreme Budget:
Court, not necessarily by way of appeal.
 Statement 1 is incorrect: Parliament can
However, the writ jurisdiction of the
Supreme Court is not exclusive. The high reduce or abolish tax but cannot increase
courts are also empowered to issue writs it.
for the enforcement of the Fundamental  Statement 2 is incorrect: The Department
Rights. of economic affairs is responsible for the
preparation of the budget.
53. Correct Option: (a)  Statement 3 is incorrect: Passing of a
Explanation: Finance bill completes the process of
enactment of the budget. The Finance
The Advocate General: Bill is introduced to give effect to the
 Statement 1 is correct: The advocate financial proposals of the Government of

RE
general is appointed by the governor. India for the following year. It is subjected
He must be a person who is qualified to to all the conditions applicable to a
be appointed a judge of a high court. Money Bill. Unlike the Appropriation Bill,
The term of office of the advocate general the amendments (seeking to reject or
is not fixed by the Constitution. Further, reduce a tax) can be moved in the case of
the Constitution does not contain the finance bill. According to the Provisional
procedure and grounds for his removal. Collection of Taxes Act of 1931, the
O
He holds office during the pleasure of Finance Bill must be enacted (i.e., passed
the governor. This means that he may be by the Parliament and assented to by the
removed by the governor at any time. He president) within 75 days. The Finance
may also quit his office by submitting
Act legalizes the income side of the
SC
his resignation to the governor.
budget and completes the process of the
Conventionally, he resigns when the
enactment of the budget.
government (council of ministers)
resigns or is replaced, as he is appointed  Statement 4 is incorrect: The
on its advice. Constitution refers to the budget as the
 Statement 2 is incorrect: Although ‘annual financial statement’. In other
the Members of the State Election words, the term ‘budget’ has nowhere
GS

Commission are appointed by the been used in the Constitution. It is the


governor, they can be removed only popular name for the ‘annual financial
by the President (and not by the statement’ that has been dealt with in
governor). Article 112 of the Constitution.
 Statement 3 is incorrect: Although the
chairman and members of a SPSC are 55. Correct Option: (a)
appointed by the governor, they can be Explanation:
removed only by the President (and not
A person cannot be a member of both
by the governor). houses of parliament at the same time.
 Statement 4 is incorrect: Although  Statement 1 is correct: If sitting member
the chairperson and members of a of one house is elected to the other
State Human Rights Commission are house, his seat in the first house becomes
appointed by the governor, they can vacant.
be removed only by the President (and  If a person is elected to two seats in
not by the governor).The President can a house, he should excuse his option
remove them on the same grounds and for one, otherwise both seats becomes
in the same manner as he can remove the vacant.
chairperson or a member of the National  Statement 2 is correct: If person is
Human Rights Commission. elected to both the houses of parliament

24
Google it:- PDF4Exams GS SCORE
https://t.me/pdf4exams «Downloaded From» https://t.me/testseries4exams

and in default of intimation his seat in manner as may be specified in rules


Rajya Sabha becomes vacant. to be made by the President. Further,
 Statement 3 is incorrect: A person cannot the validity of an order or instrument
be a member of both the Parliament and which is so authenticated shall not be
the state legislature at the same time. If a called in question on the ground that
person is so elected, his seat in Parliament it is not an order or instrument made
becomes vacant if he does not resign his or executed by the President.
seat in the state legislature within 14  The President shall make rules for the
days. more convenient transaction of the
 Statement 4 is incorrect: A person business of the Government of India,
cannot be a member of both Houses and for the allocation among Ministers
of the Legislature of a State. If they are of the said business.
elected to both the Houses, they should
immediately consult the Secretaries of  Statement 4 is correct: Article 78: It
the Houses in order to ascertain how they shall be the duty of the Prime Minister:
should vacate their seat in one House or  To communicate to the President all
the other. They will have to resign their decisions of the council of ministers
seat in one of the Houses according to

RE
relating to the administration of the
the provisions made in this behalf under affairs of the Union and proposals for
Article 190(1) of the Constitution. legislation;
 To furnish such information relating to
56. Correct Option: (d) the administration of the affairs of the
Explanation: Union and proposals for legislation as
the President may call for; and
Articles of the Indian Constitution:

O
Statement 1 is correct: Article 74: There
shall be a council of ministers with the
 If the President so requires, to submit
for the consideration of the council
of ministers any matter on which a
Prime Minister at the head to aid and
decision has been taken by a minister
SC
advise the President who shall, in the
but which has not been considered by
exercise of his functions, act in accordance
with such advice. However, the President the council.
may require the council of ministers to
reconsider such advice and the President 57. Correct Option: (c)
shall act in accordance with the advice
tendered after such reconsideration. Explanation:
GS

 (Statement 2 is correct): Article 75: Zero hour and calling attention motion:
 The Prime Minister shall be appointed  Statement 1 is correct: Zero hour is an
by the President and the other ministers informal device available to Members of
shall be appointed by the president on Parliament to raise matter without any
the advice of the Prime Minister; prior notice whereas calling attention
 The ministers shall hold office during motion used to call the attention of
the pleasure of the president; and minister to a matter of urgent public
importance.
 The council of ministers shall be
collectively responsible to the House  Zero hour:
of the People.  Statement 2 is incorrect: Unlike the
 Statement 3 is correct: Article 77: question hour, the zero hour is not
Conduct of Business of the Government mentioned in the Rules of Procedure.
of India. Thus it is an informal device available
to the members of the Parliament to
 All executive action of the Government raise matters without any prior notice.
of India shall be expressed to be taken Calling attention motion is mentioned
in the name of the President. in the rule of procedure, not Zero
 Orders and other instruments made Hour.
and executed in the name of the  The zero hour starts immediately
President shall be authenticated in such after the question hour and lasts

GS SCORE Google it:- PDF4Exams


25
https://t.me/pdf4exams «Downloaded From» https://t.me/testseries4exams

until the agenda for the day (, i.e., Indian citizens and their property in any
regular business of the House) is part of the world.
taken up. In other words, the time
gap between the question hour and 59. Correct Option: (d)
the agenda is known as zero hour.
It is an Indian innovation in the field Explanation:
of parliamentary procedures and has
The duties and functions of the CAG:
been existence since 1962.
 He audits the accounts related to all
 Calling Attention Motion: expenditure from the Consolidated Fund
 It is introduced in the Parliament of India, consolidated fund of each state
by a member to call the attention and consolidated fund of each union
of a minister to a matter of urgent territory having a Legislative Assembly.
public importance, and to seek an  He audits all expenditure from the
authoritative statement from him Contingency Fund of India and the Public
on that matter. Account of India as well as the contingency
 Statement 3 is correct: Like the zero fund of each state and the public account
hour, it is also an Indian innovation in of each state.

RE
the parliamentary procedure and has  Statement 4 is correct: He audits
been in existence since 1954. However, all trading, manufacturing, profit and
unlike the zero hour, it is mentioned loss accounts, balance sheets and
in the Rules of Procedure. other subsidiary accounts kept by any
department of the Central Government
58. Correct Option: (a) and state governments.

Explanation:
O  He audits the receipts and expenditure of
the Centre and each state to satisfy himself
The laws of Parliament are not applicable that the rules and procedures in that
in the following areas: behalf are designed to secure an effective
check on the assessment, collection and
SC
 Statement 1 is correct: The President proper allocation of revenue.
can make regulations for the peace,
progress and good government of the  Statement 1 is correct: He audits
four Union Territories-the Andaman the receipts and expenditure of the
and Nicobar Islands, Lakshadweep, Dadra following:
and Nagar Haveli and Daman and Diu. A  All bodies and authorities substantially
regulation so made has the same force and financed from the Central or state
effect as an act of Parliament. It may also
GS

revenues.
repeal or amend any act of Parliament
in relation to these union territories.  Government companies.
 Other corporations and bodies, when
 Statement 2 is correct: The governor
so required by related laws.
is empowered to direct that an act
of Parliament does not apply to a  He audits all transactions of the Central
scheduled area in the state or apply and state governments related to debt,
with specified modifications and sinking funds, deposits, advances,
exceptions. suspense accounts and remittance
business. He also audits receipts, stock
 The Governor of Assam may likewise direct accounts and others, with approval of
that an act of Parliament does not apply to the President, or when required by the
a tribal area (autonomous district) in the
President.
state or apply with specified modifications
and exceptions. The President enjoys the  He audits the accounts of any other
same power with respect to tribal areas authority when requested by the President
(autonomous districts) in Meghalaya, or Governor. For example, the audit of
Tripura and Mizoram. local bodies.
 Statement 3 is incorrect: The Parliament  He advises the President with regard to
alone can make ‘extra-territorial prescription of the form in which the
legislation’. Thus, the laws of the accounts of the Centre and the states
Parliament are also applicable to the shall be kept (Article 150).

26
Google it:- PDF4Exams GS SCORE
https://t.me/pdf4exams «Downloaded From» https://t.me/testseries4exams

 He submits his audit reports relating to  Statement 3 is incorrect: President does


the accounts of the Centre to President, not have any powers in the Enlargement
who shall, in turn, place them before both of the jurisdiction of the Supreme Court.
the Houses of Parliament (Article 151).
 He submits his audit reports relating to 61. Correct Option: (d)
the accounts of a state to governor, who Explanation:
shall, in turn, place them before the state
legislature (Article 151). Chairman of the Rajya Sabha:

 Statement 3 is correct: He ascertains  Statement 1 is correct: As Chairman of


and certifies the net proceeds of any tax Rajya Sabha, the Vice President is the
final authority on the interpretation of the
or duty (Article 279). His certificate is final.
Constitution and the Rules of Procedure
The ‘net proceeds’ means the proceeds
for all house-related matters.
of a tax or a duty minus the cost of
collection.  Statement 2 is correct: His rulings
constitute binding precedent. He also
 He acts as a guide, friend and philosopher determines whether a Rajya Sabha member
of the Public Accounts Committee of the stands to be disqualified on grounds of

RE
Parliament. defection. Such powers make him an
 He compiles and maintains the accounts important stakeholder in the functioning
of state governments. In 1976, he was of our parliamentary democracy.
relieved of his responsibilities with regard  The Vice President is also vested with
to the compilation and maintenance of powers to improve the functioning of the
accounts of the Central Government due O Upper House.
to the separation of accounts from audit,  There have been several instances where
that is, departmentalization of accounts. the current Vice President has used his
 Statement 2 is correct: He audit the powers to address issues ranging from
accounts of Panchayati Raj Institutions improving the productivity of question
SC
and Urban Local Bodies but with the hour, reducing prolonged disruptions,
permission of the State government. maintaining decorum in the House, to
facilitating discussion on issues of national
importance.
60. Correct Option: (c)
 Statement 3 is correct: The Chairman
Explanation: is the guardian of these privileges and
can also issue warrants to execute the
Enlargement of the jurisdiction of the
GS

orders of the House, where necessary.


Supreme Court: The Chairman’s consent is required to
 Statement 1 is correct: The Supreme raise a question of breach of privilege.
Court shall have such further jurisdiction He also has the discretion whether to
and powers with respect to any of the refer it to the Privileges Committee,
and whether to accept the committee’s
matters in the Union List as Parliament
recommendations.
may by law confer.
Statement 4 is correct: The Supreme

62. Correct Option: (a)
Court shall have such further jurisdiction,
and powers with respect to any matter Explanation:
as the Government of India and the Functions of the Union Executives:
Government of any State may by
special agreement confer, if Parliament  Formulation of policy as well as carrying it
by law provides for the exercise of such into execution.
jurisdiction and powers by the Supreme  Statement 1 is correct: The initiation of
Court. legislation.
 Statement 2 is incorrect: State legislature  The maintenance of order.
does not have any powers in the  Statement 2 is correct: The promotion
Enlargement of the jurisdiction of the and implementation of welfare issues
Supreme Court. of economy and social issues.

GS SCORE Google it:- PDF4Exams


27
https://t.me/pdf4exams «Downloaded From» https://t.me/testseries4exams

 The direction of foreign policy.  A Financial Bill may only be introduced in


Lok Sabha, on the recommendation of the
 Carrying or supervising of the general
President. The Bill must be passed by both
administration of the state.
Houses of Parliament, after the President
 It includes political and diplomatic has recommended that it be taken up for
activities, the recognition of a “Ruler” for consideration.
the purposes of article 366(22).
 Statement 2 is incorrect: Compensatory
 Statement 3 is incorrect: The state Afforestation Fund Bill, 2015 - Financial
legislature has “in normal circumstances” Bill.
exclusive powers to make laws with
 Statement 3 is incorrect: President’s
respect to any of the matters enumerated
(Emoluments and) Pension Act - Financial
in the State List.
Bill.
 Statement 4 is incorrect: GST -
63. Correct Option: (b) Constitutional Amendment Bill.
Explanation:
Money Bill: 64. Correct Option: (c)

RE
 A Bill is said to be a Money Bill if it only Explanation:
contains provisions related to taxation, Election of the President:
borrowing of money by the government,
expenditure from or receipt to the  Statement 1 is correct: The process
Consolidated Fund of India. Bills that only of election of the President of India
contain provisions that are incidental to is original and no other Constitution
O
these matters would also be regarded as contains a similar procedure.
Money Bills.  The question was considerably debated
 A Money Bill may only be introduced in the Constituent Assembly. It was
argued by many members that the
SC
in Lok Sabha, on the recommendation
of the President. It must be passed in Electoral College consisting of the elected
Lok Sabha by a simple majority of all members of Central Legislature as well as
members present and voting. Following those of the Legislative Assemblies of the
this, it may be sent to the Rajya Sabha States was not sufficiently representative
for its recommendations, which Lok of Peoples’ will.
Sabha may reject if it chooses to. If such  Some members, therefore, favoured
recommendations are not given within
GS

the system of direct election by the


14 days, it will deemed to be passed by people instead of an indirect round-
Parliament. about method, because such a system
 Statement 1 is correct: The Aadhaar would be most democratic and it would
(Targeted Delivery of Financial and other make the President a direct choice of the
Subsidies, benefits and services) Act, nation. This was, however, not accepted.
2016 is a money bill of the Parliament The main reasons which influenced the
of India. It aims to provide legal backing deliberations of the Constituent Assembly
to the Aadhaar unique identification for determining indirect Presidential
number project. It was passed on 11 election are:
March 2016 by the Lok Sabha.  In a country following the Cabinet
system of Government, the office of
Finance Bill:
titular Chief Executive is a technical
 A Bill that contains some provisions one, to the extent that its duties are
related to taxation and expenditure, and largely prescribed by other authorities
additionally contains provisions related (usually by the Legislature), which
to any other matter is called a Financial requires specific competence for
Bill. Therefore, if a Bill merely involves the performance of its duties from
expenditure by the government, and the incumbent. Very few voters can
addresses other issues, it will be a financial be competent to judge wisely of the
bill. technical abilities of the candidates

28
Google it:- PDF4Exams GS SCORE
https://t.me/pdf4exams «Downloaded From» https://t.me/testseries4exams

for any particular office of this type, of state and pension is charged on the
having specific, limited and defined Consolidated fund of India.
functions.
 Statement 1 is correct: Number of
 If the direct election of the President judges in the Supreme Court can be
were adopted, the Presidential increased by the parliament whereas
candidate who has to carry on an number of judges in the High Court can
election campaign from one corner of be increased by the President.
the country to another will certainly
be put up by some party or the other,  Statement 2 is correct: Impeachment
which may cause political excitement process of judge of the Supreme Court
and generate party feelings. Thus the and High court is same.
man elected to the Presidential office  Statement 3 is correct Salaries, allowances,
through this means will never be able
pensions and leaves of the judges of
to forget his party affiliations.
the Supreme Court and high court are
 As India is almost a sub-continent with determined by the Parliament.
crores of enfranchised citizens, it would
be impossible to provide an electoral

RE
machinery for the purpose of smooth
66. Correct Option: (d)
and successful Presidential election. Explanation:
 A directly elected Chief Executive may Parliamentary terms:
not be content with his position of a
mere constitutional head and can claim  Statement 2 is correct: Crossing the
to derive his authority directly from the
O floor: Passing between the member
people. So, if he wanted to assume real addressing the House and the Chair which
power, it would lead to a constitutional is considered breach of Parliamentary
deadlock and an inevitable clash with etiquette.
the Cabinet or real executive. This
Statement 3 is correct: Subordinate
SC

would definitely produce a confusion
Legislation: Rules, regulations, orders,
of responsibility.
schemes, bye-laws, etc. having the force
 Statement 2 is correct: All doubts and of law, framed by the Executive or other
disputes in connection with election subordinate authority in pursuance
of the President are inquired into and of the power conferred on it by the
decided by the Supreme Court whose Constitution or delegated to it by an Act
decision is final.
GS

of Parliament.
 The election of a person as President
 Statement 1 is correct: Whips: The office
cannot be challenged on the ground
of ‘whip’ is mentioned neither in the
that the Electoral College was incomplete
Constitution of India nor in the Rules
(i.e., existence of any vacancy among the
members of electoral college). of the House nor in a Parliamentary
Statute. It is based on the conventions
 If the election of a person as President of the parliamentary government. Every
is declared void by the Supreme Court, political party, whether ruling or Opposition
acts done by him before the date of such has its own whip in the Parliament. He is
declaration of the Supreme Court are not
appointed by the political party to serve
invalidated (i.e., they continue to remain
as an assistant floor leader. He is charged
in force).
with the responsibility of ensuring the
attendance of his party members in large
65. Correct Option: (a) numbers and securing their support in
Explanation: favour of or against a particular issue. He
regulates and monitors their behaviour
Supreme Court and High Court: in the Parliament. The members are
 Statement 4 is incorrect: Salaries and supposed to follow the directives given
allowances of the judges of the high court by the whip. Otherwise, disciplinary action
are charged on the Consolidated Fund can be taken.

GS SCORE Google it:- PDF4Exams


29
https://t.me/pdf4exams «Downloaded From» https://t.me/testseries4exams

67. Correct Option: (d) 69. Correct Option: (a)


Explanation: Explanation:
Prior recommendation of the President Parliamentary Proceedings:
is required in the following bills to be
 Statement 3 is incorrect: In the absence
introduced in the parliament:
of speaker and deputy speaker, deputy
 Statement 1 is correct: Bills related chairman of Rajya Sabha presides over a
to making changes in areas of existing joint sitting of two Houses.
states.
 Joint sitting is an extraordinary machinery
 Statement 2 is correct: Bills related to provided by the Constitution to resolve a
abolition of any tax in which states are deadlock between the two Houses over
interested. the passage of a bill.
 Statement 3 is correct: A bill which would  A deadlock is deemed to have taken place
involve expenditure from Consolidated under any one of the following three
fund of India even though it may not be situations after a bill has been passed by
a money bill. one House and transmitted to the other

RE
 Statement 4 is correct: State bills House:
imposing restrictions upon the freedom  If the bill is rejected by the other
of trade. House.
 If the Houses have finally disagreed as
68. Correct Option: (b) to the amendments to be made in the
Explanation: bill.
O
Contingency fund of India:  If more than six months have elapsed
from the date of the receipt of the bill
 Statement 1 is incorrect: It aims to by the other House without the bill
facilitate meeting of urgent unforeseen being passed by it.
SC
expenditure by the government, pending
authorization from Parliament.  Statement 2 is correct: In the above three
situations, the President can summon
 The Constitution authorized the Parliament both the Houses to meet in a joint sitting
to establish a ‘Contingency Fund of India’, for the purpose of deliberating and voting
into which amounts determined by law on the bill. It must be noted here that the
are paid from time to time. Accordingly, provision of joint sitting is applicable
the Parliament enacted the contingency
GS

to ordinary bills or financial bills only


fund of India Act in 1950. and not to money bills or Constitutional
 This fund is placed at the disposal of the amendment bills.
president, and he can make advances  In the case of a money bill, the Lok
out of it to meet unforeseen expenditure Sabha has overriding powers, while a
pending its authorization by the Constitutional amendment bill must be
Parliament. passed by each House separately.
 Statement 2 is correct: Parliamentary  Statement 1 is correct: Cut motion:
approval for such unforeseen expenditure It is a power given to the members of
is obtained ex-post-facto and an the Lok Sabha to oppose a demand
equivalent amount is drawn from in the Financial Bill discussed by the
consolidated fund of India to recoup the government. A motion may be moved to
Contingency fund of India after such an reduce the amount of a demand in any of
ex-post-facto approval. the following ways:
 Statement 3 is correct: The fund is held  Policy Cut Motion: It represents the
by the finance secretary on behalf of disapproval of the policy underlying
the President.
the demand. It states that the amount
 Statement 4 is correct: Like the public of the demand be reduced to Re 1.
account of India, it is also operated by The members can also advocate an
executive action. alternative policy.

30
Google it:- PDF4Exams GS SCORE
https://t.me/pdf4exams «Downloaded From» https://t.me/testseries4exams

 Economy Cut Motion: It represents the and members of the Union Public
economy that can be affected in the Service Commission, the governors of
proposed expenditure. It states that states, the chairman and members of
the amount of the demand be reduced finance commission, and so on.
by a specified amount (which may be
 He can seek any information relating
either a lump sum reduction in the
to the administration of affairs of the
demand or omission or reduction of
Union, and proposals for legislation
an item in the demand).
from the prime minister.
 Token Cut Motion: It ventilates a specific
 He can require the Prime Minister to
grievance that is within the sphere of
submit, for consideration of the council
responsibility of the Government of
of ministers, any matter on which a
India. It states that the amount of the
decision has been taken by a minister
demand be reduced by Rs 100.
but, which has not been considered by
the council.
70. Correct Option: (a)
 He can appoint a commission to
Explanation: investigate into the conditions of SCs,

RE
STs and other backward classes.
Executive powers and functions of the
President:  He directly administers the union
territories through administrators
 Statement 3 is incorrect: He can appoint
appointed by him.
an inter-state council to promote Centre–
state and inter- state cooperation which is  He can declare any area as scheduled
presided by Prime Minister.
O area and has powers with respect to
the administration of scheduled areas
 Statement 4 is incorrect: To make
and tribal areas.
regulations for peace, progress and good
governance of the Daman and Diu, is a
legislative power of President. 71. Correct Option: (d)
SC
 The executive powers and functions of Explanation:
the President are as follows: Consultative Committees:
 Statement 1 is correct: All executive  Consultative committees are attached
actions of the Government of India are to various ministries / departments of
formally taken in his name. the Central Government. They consist
 He can make rules specifying the of members of both the Houses of
GS

manner in which the orders and other Parliament. The Minister / Minister of
instruments made and executed in his State in charge of the Ministry concerned
name shall be authenticated. acts as the chairman of the consultative
committee of that ministry.
 Statement 2 is correct: He can make
rules for more convenient transaction  Statement 1 is correct: These
of business of the Union government, committees provide a forum for informal
and for allocation of the said business discussions between the ministers and
among the ministers. the members of Parliament on policies
and programmes of the government and
 He appoints the prime minister and the manner of their implementation.
the other ministers. They hold office
during his pleasure.  Statement 2 is correct: These committees
are constituted by the Ministry of
 He appoints the attorney general of Parliamentary Affairs.
India and determines his remuneration.
The attorney general holds office  The guidelines regarding the
during the pleasure of the President. composition, functions and procedures
of these committees are formulated by
 He appoints the comptroller and this Ministry. The Ministry also makes
auditor general of India, the chief arrangements for holding their meetings
election commissioner and other both during the session and the inter-
election commissioners, the chairman session period of Parliament.

GS SCORE Google it:- PDF4Exams


31
https://t.me/pdf4exams «Downloaded From» https://t.me/testseries4exams

 The membership of these committees 73. Correct Option: (a)


is voluntary and is left to the choice of
Explanation:
the members and the leaders of their
parties. The maximum membership of a Leader of the Opposition:
committee is 30 and the minimum is 10.
 Statement 1 is correct: In each House
 Statement 3 is correct: These committees of Parliament, there is the ‘Leader of the
are normally constituted after the new Opposition’. The leader of the largest
Lok Sabha is constituted, after General opposition party having not less than
Elections for the Lok Sabha. In other one-tenth seats of the total strength of
words, these committees shall stand the House is recognized as the leader of
dissolved upon dissolution of every Lok the Opposition in that House.
Sabha and shall be reconstituted upon
constitution of each Lok Sabha.  Statement 2 is incorrect and 3 is correct:
In a parliamentary system of government,
 Separate Informal Consultative the leader of the opposition has a
Committees of the members of Parliament significant role to play. His main functions
are also constituted for all the Railway are to provide a constructive criticism of the
Zones. Members of Parliament belonging policies of the government and to provide

RE
to the area falling under a particular an alternative government. Therefore, the
Railway Zone are nominated on the leader of Opposition in the Lok Sabha
Informal Consultative Committee of that and the Rajya Sabha were accorded
Railway Zone. statutory recognition in 1977. They are
 Unlike the Consultative Committees also entitled to the salary, allowances
attached to various ministries / and other facilities equivalent to that
departments, the meetings of the
O of a cabinet minister.
Informal Consultative Committees are to  It was in 1969 that an official leader of
be arranged during the session periods the opposition was recognised for the
only. first time. The same functionary in USA is
SC
known as the ‘minority leader’.
72. Correct Option: (d)  The British political system has a unique
Explanation: institution called the ‘Shadow Cabinet’.
It is formed by the Opposition party to
Lok Adalat: balance the ruling cabinet and to prepare
 Statement 1 is correct: It is presided over its members for future ministerial offices.
by a sitting or retired judicial officer as In this shadow cabinet, almost every
GS


the chairman member in the ruling cabinet is
 Statement 2 is correct: Its decision is ‘shadowed’ by a corresponding member
binding on parties to the dispute and in the opposition cabinet. This shadow
its order is capable of execution through cabinet serves as the ‘alternate cabinet’
legal process. if there is change of government. That is
why Ivor Jennings described the leader
 Statement 3 is correct: It deals in matters
of Opposition as the ‘alternative Prime
related to both civil and criminal cases.
Minister’. He enjoys the status of a minister
 Statement 4 is correct: There is no court and is paid by the government.
fee. If the case is already filed in the regular
court, the fee paid will be refunded if the
74. Correct Option: (d)
dispute is settled at the Lok Adalat.
Explanation:
 Lok Adalats (people’s courts) settle dispute
through conciliation and compromise. Ordinance-Making Power of the
 The main thrust of Lok Adalats is on President:
compromise. When no compromise is  Statement 1 is incorrect: President’s
reached, the matter goes back to the power of ordinance-making is not
court. While conducting the proceedings; a discretionary power, and he can
Lok Adalat acts as a conciliator and not as promulgate or withdraw an ordinance only
an arbitrator. on the advice of the council of ministers

32
Google it:- PDF4Exams GS SCORE
https://t.me/pdf4exams «Downloaded From» https://t.me/testseries4exams

headed by the Prime Minister. ceases to operate on the expiry of


 Statement 2 is incorrect: He can six weeks from the reassembly of
promulgate an ordinance only when Parliament.
both the Houses of Parliament are not  An ordinance like any other legislation,
in session or when either of the two can be retrospective, that is, it may
Houses of Parliament is not in session. An come into force from a back date.
ordinance can also be issued when only It may modify or repeal any act of
one House is in session because a law can Parliament or another ordinance. It can
be passed by both the Houses and not
alter or amend a tax law also. However,
by one House alone. An ordinance made
it cannot be issued to amend the
when both the Houses are in session is
void. Thus, the power of the President to Constitution.
legislate by ordinance is not a parallel
power of legislation. 75. Correct Option: (d)
 Statement 3 is incorrect: the maximum Explanation:
life of an ordinance can be six months
and six weeks. Ad hoc Judge:

RE
 Article 123 of the Constitution empowers  Statement 1 is correct: When there is a
the President to promulgate ordinances lack of quorum of the permanent judges
during the recess of Parliament. to hold or continue any session of the
 These ordinances have the same force Supreme Court, the Chief Justice of India
and effect as an act of Parliament, but can appoint a judge of a High Court as
are in the nature of temporary laws.
O an ad hoc judge of the Supreme Court
for a temporary period.
 The ordinance-making power is the most
important legislative power of the  Statement 2 is correct: He can do so only
President. It has been vested in him to after consultation with the Chief Justice of
deal with unforeseen or urgent matters. the High Court concerned and with the
SC
But, the exercises of this power is subject previous consent of the President.
to the following limitations:
 Statement 3 is correct: The judge
 He can make an ordinance only when so appointed should be qualified for
he is satisfied that the circumstances appointment as a judge of the Supreme
exist that render it necessary for him to Court. It is the duty of the judge so
take immediate action. appointed to attend the sittings of the
 His ordinance-making power is Supreme Court, in priority to other duties
GS

coextensive as regards all matters of his office. While so attending, he


except duration, with the law-making enjoys all the jurisdiction, powers and
powers of the Parliament. This has two privileges (and discharges the duties)
implications: of a judge of the Supreme Court.
 An ordinance can be issued only
on those subjects on which the 76. Correct Option: (c)
Parliament can make laws.
Explanation:
 An ordinance is subject to the same
constitutional limitation as an act Finance Commission:
of Parliament. Hence, an ordinance  Statement 1 is incorrect: President
cannot abridge or take away any of appoints Attorney General of India
the fundamental rights. who holds office during the pleasure
 Every ordinance issued by the President of President. However, Comptroller
during the recess of parliament must & Auditor General of India though
be laid before both the Houses of appointed by the President does not
Parliament when it reassembles. If the hold the office during the pleasure of
ordinance is approved by both the President. He can be removed by the
Houses, it becomes an act. If Parliament President on same grounds and in the
takes no action at all, the ordinance same manner as a judge of the Supreme

GS SCORE Google it:- PDF4Exams


33
https://t.me/pdf4exams «Downloaded From» https://t.me/testseries4exams

Court. In other words, he can be removed  Statement 3 is incorrect: Governor


by the president on the basis of a resolution should establish a Regional Council for the
passed to that effect by both the Houses Tuensang district and make rules for the
of Parliament with special majority either compositions, qualifications and service
on the ground of proved misbehaviour or conditions of the member of council.
incapacity. This provision accounts for the state
of Nagaland and not for Arunachal
 Statement 2 is incorrect: President can Pradesh.
address Parliament at the commencement
 Under Article 371-H, the Constitution
of the first session after each general
made the special provision for Arunachal
election and first session of each year.
Pradesh, by assigning Governor of
 Statement 3 is correct: Article 280 of Arunachal Pradesh special responsibilities
the Constitution of India provides for a for law and order in the state. The
Finance Commission as a quasi-judicial Arunachal Pradesh Legislative Assembly
body. It is constituted by the President is to consist of not less than 30 members.
of India every fifth year or at such earlier
time as he considers necessary. 78. Correct Option: (d)

RE
 The Finance Commission is required to Explanation:
make recommendations to the President
of India on the following matters:
Territorial Extent of Central and State
Legislation
 The distribution of the net proceeds of
The Constitution defines the territorial limits
taxes to be shared between the Centre
of the legislative powers vested in the Centre
and the states, and the allocation and the states in the following way:
O
between the states of the respective
shares of such proceeds.  Statement 1 is correct: The Parliament
can make laws for the whole or any part
 The principles that should govern of the territory of India. The territory
the grants-in-aid to the states by the of India includes the states, the union
SC
Centre (i.e., out of the consolidated territories, and any other area for the time
fund of India). being included in the territory of India.
 The measures needed to augment  Statement 2 is correct: A state legislature
the consolidated fund of a state to can make laws for the whole or any part
supplement the resources of the of the state. The laws made by a state
panchayats and the municipalities legislature are not applicable outside the
in the state on the basis of the state, except when there is a sufficient
GS

recommendations made by the state nexus between the state and the object.
finance commission.  Statement 3 is correct: The Parliament
 Any other matter referred to it by the alone can make ‘extra-territorial
legislation’. Thus, the laws of the
president in the interests of sound
Parliament are also applicable to the
finance.
Indian citizens and their property in any
part of the world.
77. Correct Option: (a)  The Constitution places certain restrictions
Explanation: on the plenary territorial jurisdiction of
the Parliament. In other words, the laws
Special Provisions for Arunanchal Pradesh of Parliament are not applicable in the
Governor: following areas:
 Statement 1 is incorrect: In discharge  The President can make regulations
of his responsibilities Governor does for the peace, progress and good
not exercises his individual judgment government of the four Union
independent of Council of Ministers. Territories—the Andaman and Nicobar
Islands, Lakshadweep, Dadra and
 Statement 2 is correct: This special Nagar Haveli and Daman and Diu. A
responsibility of Governor Cease when regulation so made has the same force
the President so directs. and effect as an act of Parliament. It

34
Google it:- PDF4Exams GS SCORE
https://t.me/pdf4exams «Downloaded From» https://t.me/testseries4exams

may also repeal or amend any act of must continue to be in English only
Parliament in relation to these union (until Parliament otherwise provides).
territories.
 Similarly, a state legislature can
 The governor is empowered to direct prescribe the use of any language (other
that an act of Parliament does not than English) with respect to bills, acts,
apply to a scheduled area in the state ordinances, orders, rules, regulations
or apply with specified modifications or bye-laws, but a translation of the
and exceptions. same in the English language is to be
published.
 The Governor of Assam may likewise
direct that an act of Parliament  Statement 1 is correct: The Official
does not apply to a tribal area Language Act of 1963 lays down that
(autonomous district) in the state or Hindi translation of acts, ordinances,
apply with specified modifications and orders, regulations and bye-laws
exceptions. The President enjoys the published under the authority of
same power with respect to tribal areas the president are deemed to be
(autonomous districts) in Meghalaya, authoritative texts.
Tripura and Mizoram.  Every bill introduced in the Parliament

RE
is to be accompanied by a Hindi
79. Correct Option: (a) translation. Similarly, there is to be
a Hindi translation of state acts or
Explanation: ordinances in certain cases.
Language of the Judiciary and Texts of  The act also enables the governor of
Laws: O a state, with the previous consent of
 Statement 2 is incorrect: Governor can the President, to authorize the use of
authorize the use of Hindi or any other Hindi or any other official language of
official language of the state for the the state for judgements, decrees and
judgment, decrees and orders passed by orders passed by the high court of the
state but they should be accompanied
SC
the high court of the state.
by an English translation. For example,
 Statement 3 is incorrect: The Parliament Hindi is used in Uttar Pradesh, Madhya
has not made any provision for the use of Pradesh, Bihar and Rajasthan for this
Hindi in the Supreme Court. Hence, the purpose.
Supreme Court hears only those who
petition or appeal in English.  The Parliament has not made any
provision for the use of Hindi in
 The Constitutional provisions dealing with the Supreme Court. Hence, the
GS

the language of the courts and legislation Supreme Court hears only those
are as follows: who petition or appeal in English. In
 Until Parliament provides otherwise, 1971, a petitioner insisted on arguing
the following are to be in the English in Hindi a habeas corpus petition in
language only: the Supreme Court. But, the Court
cancelled his petition on the ground
 All proceedings in the Supreme Court that the language of the Court was
and in every high court. English and allowing Hindi would be
 The authoritative texts of all bills, acts, unconstitutional.
ordinances, orders, rules, regulations
and bye-laws at the Central and state 80. Correct Option: (d)
levels.
Explanation:
 However, the governor of a state, with
the previous consent of the president, State’s consent with respect to Bills:
can authorise the use of Hindi or any  Statement 1 is incorrect: There is no time
other official language of the state, in limit for the states for their approval in
the proceedings in the high court of those provisions of the Constitution which
the state, but not with respect to the are related to federal structure of polity.
judgements, decrees and orders passed
by it. In other words, the judgements,  Statement 2 is incorrect: But there is
decrees and orders of the high court six months’ time limit is given for the

GS SCORE Google it:- PDF4Exams


35
https://t.me/pdf4exams «Downloaded From» https://t.me/testseries4exams

states approval in the Bills related to legislature to ascertain its views and
the formation of new state or changes the Parliament can itself take any
in the existing states of union of India. action as it deems fit.
 Those provisions of the Constitution
which are related to the federal 81. Correct Option: (d)
structure of the polity can be amended Explanation:
by a special majority of the Parliament
and also with the consent of half of the Provisions mentioned in the
state legislatures by a simple majority. Constitution:
 If one or some or all the remaining states  Statement 1 is incorrect: All doubts and
take no action on the bill, it does not disputes in connection with election of
matter; the moment half of the states give the Vice-President are inquired into and
their consent, the formality is completed. decided by the Supreme Court whose
There is no time limit within which the decision is final. The election of a person
states should give their consent to the as Vice-President cannot be challenged
on the ground that the electoral college
bill.
was incomplete (i.e., existence of any
Article 3 authorizes the Parliament to: vacancy among the members of Electoral

RE

College). If the election of a person as Vice-
 Form a new state by separation of
President is declared void by the Supreme
territory from any state or by uniting
Court, acts done by him before the date
two or more states or parts of states
of such declaration of the Supreme Court
or by uniting any territory to a part of are not invalidated (i.e., they continue to
any state. remain in force).
 Increase the area of any state.
O  Statement 2 is incorrect: The Constitution
 Diminish the area of any state. does not contain any specific procedure
for the selection and appointment
 Alter the boundaries of any state. of the Prime Minister. Article 75 says
only that the Prime Minister shall be
SC
 Alter the name of any state.
appointed by the president. Prime
 Article 3 lays down two conditions in minister recommends persons who
this regard: one, a bill contemplating can be appointed as ministers by the
the above changes can be introduced president. The President can appoint
in the Parliament only with the prior only those persons as ministers who are
recommendation of the President; recommended by the Prime Minister.
and two, before recommending the bill,
GS

the President has to refer the same


to the state legistature concerned for
82. Correct Option: (c)
expressing its views within a specified Explanation:
period.
Chairman of Rajya Sabha:
 The power of Parliament to form new
 The presiding officer of the Rajya
states includes the power to form a new
Sabha is known as the Chairman. The
state or union territory by uniting a part vice-president of India is the ex-officio
of any state or union territory to any other Chairman of the Rajya Sabha. During any
state or union territory. period when the Vice-President acts as
 The President (or Parliament) is not President or discharges the functions of
bound by the views of the state the President, he does not perform the
legislature and may either accept or duties of the office of the Chairman of
reject them, even if the views are received Rajya Sabha.
in time.  Statement 1 is correct: The Chairman of
 It is not necessary to make a fresh the Rajya Sabha can be removed from
reference to the state legislature every his office only if he is removed from the
time an amendment to the bill is moved office of the Vice-President.
and accepted in Parliament.  As a presiding officer, the powers and
 In case of a union territory, no reference functions of the Chairman in the Rajya
need be made to the concerned Sabha are similar to those of the Speaker

36
Google it:- PDF4Exams GS SCORE
https://t.me/pdf4exams «Downloaded From» https://t.me/testseries4exams

in the Lok Sabha. However, the Speaker  Between two or more states.
has two special powers which are not
 In the above federal disputes, the Supreme
enjoyed by the Chairman:
Court has exclusive original jurisdiction.
 The Speaker decides whether a bill is a Exclusive means, no other court can
money bill or not and his decision on decide such disputes and original means,
this question is final. the power to hear such disputes in the
 The Speaker presides over a joint first instance, not by way of appeal.
sitting of two Houses of Parliament.
 With regard to the exclusive original
 Unlike the Speaker (who is a member jurisdiction of the Supreme Court, two
of the House), the Chairman is not points should be noted. One, the dispute
a member of the House. But like the must involve a question (whether of law or
Speaker, the Chairman also cannot vote
fact) on which the existence or extent of
in the first instance. He too can cast a vote
a legal right depends. Thus, the questions
in the case of an equality of votes.
of political nature are excluded from it.
 Statement 3 is correct: The Vice- Two, any suit brought before the Supreme
President cannot preside over a sitting Court by a private citizen against the

RE
of the Rajya Sabha as its Chairman
Centre or a state cannot be entertained
when a resolution for his removal is
under this.
under consideration.
 Statement 2 is correct: However, he can  Further, this jurisdiction of the Supreme
be present and speak in the House and Court does not extend to the following:
can take part in its proceedings, without
O  A dispute arising out of any pre-
voting, even at such a time (while the Constitution treaty, agreement,
Speaker can vote in the first instance covenant, engagement, sanad or other
when a resolution for his removal is under
similar instrument.
consideration of the Lok Sabha).
 A dispute arising out of any treaty,
SC
 As in case of the Speaker, the salaries
and allowances of the Chairman are agreement, etc., which specifically
also fixed by the Parliament. They provides that the said jurisdiction does
are charged on the Consolidated Fund not extent to such a dispute.
of India and thus are not subject to the  Statement 2 is incorrect: Inter-state
annual vote of Parliament. water disputes.
 During any period when the Vice-
 Matters referred to the Finance
GS

President acts as President or discharges


Commission.
the functions of the President, he is not
entitled to any salary or allowance payable  Adjustment of certain expenses and
to the Chairman of the Rajya Sabha. But pensions between the Centre and the
he is paid the salary and allowance of the states.
President during such a time.
 Statement 1 is incorrect: Ordinary
dispute of commercial nature between
83. Correct Option: (d)
the Centre and the states.
Explanation:
 Statement 3 is incorrect: Recovery
Original Jurisdiction of Supreme Court: of damages by a state against the
 As a federal court, the Supreme Court Centre.
decides the disputes between different
units of the Indian Federation. More 84. Correct Option: (c)
elaborately, any dispute between:
Explanation:
 The Centre and one or more states; or
Reorganization of the states:
 The Centre and any state or states on
 Statement 1 is incorrect: In case of a
one side and one or more states on union territory, no reference need be
the other; or made to the concerned legislature to

GS SCORE Google it:- PDF4Exams


37
https://t.me/pdf4exams «Downloaded From» https://t.me/testseries4exams

ascertain its views and the Parliament rightly described as ‘an indestructible
can itself take any action as it deems fit. union of destructible states’
 Statement 2 is incorrect: The President
(or Parliament) is not bound by the views 85. Correct Option: (c)
of the state legislature and may either
accept or reject them, even if the views Explanation:
are received in time. Office of the President of India:
 Article 3 authorizes the Parliament to:  The President is elected not directly by the
 form a new state by separation of people but by the members of electoral
territory from any state or by uniting college consisting of:
two or more states or parts of states  The elected members of both the
or by uniting any territory to a part of Houses of Parliament.
any state,
 The elected members of the legislative
 increase the area of any state,
assemblies of the states.
 diminish the area of any state,
 The elected members of the legislative

RE
 alter the boundaries of any state, and assemblies of the Union Territories of
 alter the name of any state. Delhi and Puducherry.

 However, Article 3 lays down two  Statement 1 is incorrect: Thus, the


conditions in this regard: nominated members of both of Houses
of Parliament, the nominated members
 a bill contemplating the above changes of the state legislative assemblies, the
can be introduced in the Parliament
only with the prior recommendation
O members (both elected and nominated)
of the state legislative councils (in case
of the President; and
of the bicameral legislature) and the
 before recommending the bill, the nominated members of the Legislative
President has to refer the same to
SC
Assemblies of Delhi and Puducherry
the state legislature concerned for do not participate in the election of the
expressing its views within a specified President.
period.
 Before entering upon his office, the
 Statement 3 is correct: Such laws do not President has to make and subscribe to
require a special majority but can be an oath or affirmation. In his oath, the
passed by a simple majority. President swears:
GS

 Further, the power of Parliament to form


 To faithfully execute the office.
new states includes the power to form a
new state or union territory by uniting a  To preserve, protect and defend the
part of any state or union territory to any Constitution and the law.
other state or union territory.
 To devote himself to the service and
 Further, it is not necessary to make a fresh well-being of the people of India.
reference to the state legislature every
 Statement 3 is incorrect: The oath of
time an amendment to the bill is moved
and accepted in Parliament office to the President is administered
by the Chief Justice of India and in his
 It is thus clear that the Constitution absence, the seniormost judge of the
authorizes the Parliament to form new Supreme Court available. Any other person
states or alter the areas, boundaries or acting as President or discharging the
names of the existing states without their functions of the President also undertakes
consent. the similar oath or affirmation.
 In other words, the Parliament can redraw  Statement 4 is incorrect: The President
the political map of India according to its
holds office for a term of five years from
will.
the date on which he enters upon his
 Hence, the territorial integrity or continued office. However, he can resign from his
existence of any state is not guaranteed office at any time by addressing the
by the Constitution. Therefore, India is resignation letter to the Vice-President.

38
Google it:- PDF4Exams GS SCORE
https://t.me/pdf4exams «Downloaded From» https://t.me/testseries4exams

Further, he can also be removed from the  Statement 2 is correct: No demand


office before completion of his term by for a grant can be made except on his
the process of impeachment. recommendation.
 A person to be eligible for election as  He constitutes a finance commission
President should fulfill the following after every five years to recommend the
qualifications: distribution of revenues between the
Centre and the states.
 He should be a citizen of India.
 He should have completed 35 years of
age.
88. Correct Option: (b)
 Statement 2 is correct: He should be Explanation:
qualified for election as a member of Features of Cabinet Committees:
the Lok Sabha.
 Statement 1 is incorrect: They are
 He should not hold any office of profit extra-constitutional in emergence. In
under the Union government or any other words, they are not mentioned in
state government or any local authority the Constitution. However, the Rules of
or any other public authority. Business provide for their establishment.

RE
 They are of two types—standing and ad
86. Correct Option: (a) hoc. The former are of a permanent nature
Explanation: while the latter are of a temporary nature.
The ad hoc committees are constituted
Representative Democracy: from time to time to deal with special
 Statement 1 is correct: Legislation may
O problems. They are disbanded after their
be initiated by stakeholders or from within task is completed.
the government.  They are set up by the Prime Minister
 Statement 2 is incorrect: Public according to the exigencies of the time
participation regarding draft Bills and requirements of the situation.
SC
is not statutorily mandated. The Hence, their number, nomenclature, and
government has taken steps to increase composition varies from time to time.
public engagement. In some cases the  Their membership varies from three to
government has invited stakeholders to eight. They usually include only Cabinet
consultations on proposed Bills. Ministers. However, the non-cabinet
 For instance the Department of Ministers are not debarred from their
Information Technology had invited membership.
GS

experts for consultations on the draft  Statement 2 is correct: They not only
Electronic Service Delivery Bill, 2011. The include the Ministers in charge of
Ministry of Finance had held regional subjects covered by them but also include
consultations over the draft direct tax other senior Ministers.
code.
 They are mostly headed by the Prime
Minister. Sometimes other Cabinet
87. Correct Option: (b) Ministers, particularly the Home Minister
or the Finance Minister, also acts as their
Explanation:
Chairman. But, in case the Prime Minister
Financial Powers of the president: is a member of a committee, he invariably
presides over it.
 Statement 1 is incorrect: He can make
advances out of the Contingency  Statement 3 is correct: They not only sort
fund of India to meet any unforeseen out issues and formulate proposals for
expenditure. the consideration of the Cabinet, but also
take decisions. However, the Cabinet can
 Statement 3 is correct: Money bills can
review their decisions.
be introduced in the Parliament only with
his prior recommendation.  They are an organizational device to
 He causes to be laid before the Parliament reduce the enormous workload of the
the annual financial statement. (i.e, the Cabinet. They also facilitate in-depth
Union Budget). examination of policy issues and effective

GS SCORE Google it:- PDF4Exams


39
https://t.me/pdf4exams «Downloaded From» https://t.me/testseries4exams

coordination. They are based on the  He recommends persons who can


principles of division of labour and be appointed as ministers by the
effective delegation. President. The President can appoint
only those persons as ministers who
are recommended by the Prime
89. Correct Option: (d)
Minister.
Explanation:  He allocates and reshuffles various
Pardoning Power of the Parliament: portfolios among the ministers.

 The Supreme Court examined the  Statement 1 is correct: He can ask


pardoning power of the President under a minister to resign or advise the
different cases and laid down the following President to dismiss him in case of
principles: difference of opinion.
 He presides over the meeting of
 Statement 1 is correct: The petitioner
council of ministers and influences its
for mercy has no right to an oral
decisions.
hearing by the President.
 He guides, directs, controls, and

RE
 The President can examine the evidence
coordinates the activities of all the
afresh and take a view different from
ministers.
the view taken by the court.
 He can bring about the collapse of the
 Statement 2 is correct: The power is co-uncil of ministers by resigning from
to be exercised by the President on office.
the advice of the union cabinet.
O  In Relation to Parliament: The Prime
 Statement 3 is correct: The President Minister is the leader of the Lower House.
is not bound to give reasons for his In this capacity, he enjoys the following
order. powers:
SC
 The President can afford relief not only  He advises the President with regard
from a sentence that he regards as to summoning and proroguing of the
unduly harsh but also from an evident sessions of the Parliament.
mistake.  Statement 2 is correct: He can
 There is no need for the Supreme Court recommend dissolution of the Lok
to lay down specific guidelines for the Sabha to President at any time.
exercise of power by the President.
GS

 He announces government policies on


 The exercise of power by the President the floor of the House.
is not subject to judicial review except  Other Powers & Functions: In addition
where the presidential decision to the above-mentioned three major
is arbitrary, irrational, malafide or roles, the Prime Minister has various other
discriminatory. roles. These are:
 Where the earlier petition for mercy  He is the chairman of the Planning
has been rejected by the President, Commission (now NITI Aayog),
stay cannot be obtained by filing National Development Council,
another petition. National Integration Council, Inter-
State Council and National Water
Resources Council.
90. Correct Option: (d)
 He plays a significant role in shaping
Explanation:
the foreign policy of the country.
Powers of Prime Minister:  He is the chief spokesman of the Union
 In Relation to Council of Ministers: government.
The Prime Minister enjoys the following  Statement 3 is correct: He is the crisis
powers as head of the Union council of manager-in-chief at the political
ministers: level during emergencies.

40
Google it:- PDF4Exams GS SCORE
https://t.me/pdf4exams «Downloaded From» https://t.me/testseries4exams

 As a leader of the nation, he meets wastefulness and extravagance of such


various sections of people in different expenditure. However, unlike the legal
states and receives memoranda from and regulatory audit, which is obligatory
them regarding their problems, and so on the part of the CAG, the propriety
on. audit is discretionary.
 He is leader of the party in power.  The Constitution of India visualizes the
 He is political head of the services. CAG to be Comptroller as well as Auditor
General. However, in practice, the CAG is
fulfilling the role of an Auditor-General
91. Correct Option: (c) only and not that of a Comptroller.
Explanation:  Statement 3 is correct: In other words,
Comptroller & Auditor General (CAG): ‘the CAG has no control over the issue
of money from the Consolidated Fund
 The CAG is an agent of the Parliament and many departments are authorized to
and conducts audit of expenditure on draw money by issuing cheques without
behalf of the Parliament. Therefore, he is
specific authority from the CAG, who is
responsible only to the Parliament.
concerned only at the audit stage when

RE
 The role of CAG is to uphold the the expenditure has already taken place’.
Constitution of India and the laws
of Parliament in the field of financial  In this respect, the CAG of India differs
administration. The accountability of the totally from the CAG of Britain who has
executive (i.e., council of ministers) to powers of both Comptroller as well as
the Parliament in the sphere of financial Auditor General. In other words, in Britain,
administration is secured through audit the executive can draw money from the


reports of the CAG.
Statement 1 is correct: The CAG has
O public exchequer only with the approval
of the CAG.
more freedom with regard to audit of
expenditure than with regard to audit 92. Correct Option: (a)
SC
of receipts, stores and stock. “Whereas
Explanation:
in relation to expenditure he decides the
scope of audit and frames his own audit Adjournment Motion:
codes and manuals, he has to proceed with
 Statement 1 is correct: It is introduced
the approval of the executive government
in relation to rules for the conduct of the in the Parliament to draw attention of the
other audits.” House to a definite matter of urgent
GS

public importance, and needs the


 The CAG has ‘to ascertain whether money support of 50 members to be admitted.
shown in the accounts as having been
disbursed was legally available for and  Statement 2 is incorrect: It involves
applicable to the service or the purpose to an element of censure against the
which they have been applied or charged government and hence Rajya Sabha
and whether the expenditure conforms to is not permitted to make use of this
the authority that governs it’. device.
 In addition to this legal and regulatory  As it interrupts the normal business of the
audit, the CAG can also conduct the House, it is regarded as an extraordinary
propriety audit, that is, he can look into device.
the ‘wisdom, faithfulness and economy’
 The discussion on an adjournment motion
of government expenditure and comment
should last for not less than two hours
on the wastefulness and extravagance of
and thirty minutes.
such expenditure.
 The right to move a motion for an
 Statement 2 is incorrect: In addition to
the legal and regulatory audit, the CAG adjournment of the business of the House
can also conduct the propriety audit, is subject to the following restrictions:
that is, he can look into the ‘wisdom,  It should raise a matter which is
faithfulness and economy’ of government definite, factual, urgent and of public
expenditure and comment on the importance.

GS SCORE Google it:- PDF4Exams


41
https://t.me/pdf4exams «Downloaded From» https://t.me/testseries4exams

 It should not cover more than one their ministries/departments as cabinet


matter. ministers do. However, they are not
 It should be restricted to a specific members of the cabinet and do not
matter of recent occurrence and should attend the cabinet meetings unless
not be framed in general terms. specially invited when something related
to their ministries/departments are
 It should not raise a question of considered by the cabinet.
privilege.
 The ministers of state can either be
 It should not revive discussion on a given independent charge of ministries/
matter that has been discussed in the departments or can be attached to
same session.
cabinet ministers. In case of attachment,
 It should not deal with any matter that they may either be given the charge of
is under adjudication by court. departments of the ministries headed by
 It should not raise any question that the cabinet ministers or allotted specific
can be raised on a distinct motion. items of work related to the ministries
headed by cabinet ministers. In both the
cases, they work under the supervision
93. Correct Option: (a)

RE
and guidance as well as under the overall
Explanation: charge and responsibility of the cabinet
ministers.
Composition of the Council of Ministers:
 It must also be mentioned here that
 The council of ministers consists of three
there is one more category of ministers,
categories of ministers, namely, cabinet
called parliamentary secretaries. They are
ministers, ministers of state, and deputy
O
ministers. The difference between them
the members of the last category of the
lies in their respective ranks, emoluments, council of ministers (which is also known
and political importance. At the top as the ‘ministry’). They have no department
of all these ministers stands the Prime under their control. They are attached to
SC
Minister—the supreme governing the senior ministers and assist them in the
authority of the country. discharge of their parliamentary duties.
However, since 1967, no parliamentary
 Statement 1 is correct: The Cabinet is a secretaries have been appointed except
smaller body which consists of a few
during the first phase of Rajiv Gandhi
important senior ministers. The cabinet
Government.
ministers head the important ministries
of the Central government like home,  At times, the council of ministers may
GS

defence, finance, external affairs and so also include a deputy prime minister. The
forth. They are members of the cabinet, deputy prime ministers are appointed
attend its meetings and play an important mostly for political reasons.
role in deciding policies. Thus, their
responsibilities extend over the entire
gamut of Central government. 94. Correct Option: (c)
 Statement 2 is incorrect: The Deputy Explanation:
Ministers act as assistants to their Collective Responsibility of the Council of
chiefs and look after their work in their
Minister:
absence. The number of ministers may
vary according to the volume and variety  The fundamental principle underlying
of work of the Central government. the working of parliamentary system of
 Statement 3 is incorrect: A Minister of government is the principle of collective
State is a junior minister in the Council responsibility. Article 75 clearly states
of Ministers in the Federal or Central that the council of ministers is collectively
Government who may assist a Cabinet responsible to the Lok Sabha. This
Minister or have independent charge of means that all the ministers own joint
a ministry. In case of independent charge, responsibility to the Lok Sabha for all
they perform the same functions and their acts of omission and commission.
exercise the same powers in relation to They work as a team and swim or sink
together.

42
Google it:- PDF4Exams GS SCORE
https://t.me/pdf4exams «Downloaded From» https://t.me/testseries4exams

 When the Lok Sabha passes a no-  The Attorney General is necessary for
confidence motion against the council of giving advice to the Government of
ministers, all the ministers have to resign India in legal matters referred to him. He
including those ministers who are from also performs other legal duties assigned
the Rajya Sabha. Alternatively, the council to him by the President.
of ministers can advise the president to  Statement 2 is correct: The Attorney
dissolve the Lok Sabha on the ground General has the right of audience in
that the House does not represent the all Courts in India as well as the right
views of the electorate faithfully and call to participate in the proceedings of
for fresh elections. The President may not the Parliament, though not to vote.
oblige the council of ministers that has  The Attorney General appears on behalf
lost the confidence of the Lok Sabha. of Government of India in all cases
 Statement 1 is incorrect: The Council of (including suits, appeals and other
Ministers is collectively responsible to proceedings) in the Supreme Court in
the Lok Sabha only. which Government of India is concerned.
He also represents the Government
 Statement 2 is incorrect: A vote of no- of India in any reference made by the
confidence against any minister can be President to the Supreme Court under

RE
held only in Lok Sabha. Article 143 of the Constitution.
 Statement 3 is correct: The principle of  Statement 3 is correct: The Attorney
collective responsibility also means that General is assisted by a Solicitor
the Cabinet decisions bind all cabinet General and four additional Solicitors
ministers (and other ministers) even if General. The Attorney General is to
they differed in the cabinet meeting. It
O be consulted only in legal matters
is the duty of every minister to stand by of real importance and only after
cabinet decisions and support them both the Ministry of Law has been consulted.
within and outside the Parliament. All references to the Attorney General
If any minister disagrees with a cabinet are made by the Law Ministry.
SC

decision and is not prepared to defend
it, he must resign. Several ministers 96. Correct Option: (d)
have resigned in the past owing to their
Explanation:
differences with the cabinet. For example,
Dr BR Ambedkar resigned because of his Appointment of Ministers:
differences with his colleagues on the  The Prime Minister is appointed by the
Hindu Code Bill in 1953. C D Deshmukh President, while the other ministers are
GS

resigned due to his differences on the appointed by the President on the advice
policy of reorganisation of states. Arif of the Prime Minister.
Mohammed resigned due to his opposition
to the Muslim Women (Protection of  This means that the President can appoint
only those persons as ministers who are
Rights on Divorce) Act, 1986.
recommended by the Prime minister.
Usually, the members of Parliament, either
95. Correct Option: (d) Lok Sabha or Rajya Sabha, are appointed
as ministers.
Explanation:
 Statement 1 is correct: A person who
Attorney General of India: is not a member of either House of
 Statement 1 is correct: The Attorney Parliament can also be appointed as a
General of India is the Indian minister.
government’s chief legal advisor, and is  Statement 2 is incorrect: But, within
primary lawyer in the Supreme Court six months, he must become a member
of India. He can be said to be the lawyer (either by election or by nomination) of
from government’s side. either House of Parliament, otherwise, he
ceases to be a minister.
 He is appointed by the President
of India under Article 76(1) of  Statement 3 is correct: A minister who
the Constitution and holds office during is a member of one House of Parliament
the pleasure of the President. has the right to speak and to take part

GS SCORE Google it:- PDF4Exams


43
https://t.me/pdf4exams «Downloaded From» https://t.me/testseries4exams

in the proceedings of the other House discharge their functions as entrusted


also, but he can vote only in the House of upon them by the Constitution.
which he is a member.  Statement 1 is incorrect: They have
freedom of speech in Parliament. No
97. Correct Option: (d) member is liable to any proceedings in any
court for anything said or any vote given
Explanation:
by him in Parliament or its committees.
No-Confidence Motion: This freedom is subject to the provisions
 Article 75 of the Constitution says that the of the Constitution and to the rules
council of ministers shall be collectively and standing orders regulating the
responsible to the Lok Sabha. It means procedure of Parliament.
that the ministry stays in office so long as  Statement 2 is correct: They are
it enjoys confidence of the majority of the exempted from jury service. They can
members of the Lok Sabha. In other words, refuse to give evidence and appear as a
the Lok Sabha can remove the ministry witness in a case pending in a court when
from office by passing a no-confidence Parliament is in session.

RE
motion. The motion needs the support of
50 members to be admitted.  Statement 3 is correct: They cannot be
arrested during the session of Parliament
 It need not state the reasons for its and 40 days before the beginning and
adoption in the Lok Sabha. 40 days after the end of a session.
 It is moved for ascertaining the confidence  Statement 4 is incorrect: The privilege
O
of Lok Sabha in the council of ministers. of not being arrested is available only in
 Statement 2 is correct: If it is passed in civil cases and not in criminal cases or
the Lok Sabha, the council of ministers preventive detention cases.
headed by Prime Minister must resign
SC
from office. 99. Correct Option: (c)
 Statement 1 and 3 are correct: When
Explanation:
the Lok Sabha passes a no-confidence
motion against the council of ministers, all Representation of Peoples Act 1951:
the ministers have to resign including  The Representation of People Act, 1951 is
those ministers who are from the Rajya
an act of Parliament of India to provide
Sabha.
GS

for the conduct of elections of the Houses


 Statement 4 is correct: The council of of Parliament, the qualifications and
ministers can advise the President to disqualifications for membership of those
dissolve the Lok Sabha on the ground that Houses, the corrupt practices and other
the House does not represent the views of offences at or in connection with such
the electorate faithfully and call for fresh elections and the decision of doubts and
elections. The President may not oblige disputes arising out of or in connection
the council of ministers that has lost with such elections.
the confidence of the Lok Sabha.
 Option (c) is correct: The Parliament
has laid down the disqualification in
98. Correct Option: (a) Representation of Peoples Act 1951. In
Explanation: such cases President’s decision is final.
However, he should obtain the opinion
Individual Privileges enjoyed by the of the Election Commission and act
Members of the Parliament: accordingly.
 Parliamentary privilege refers to rights  In case of disqualification on grounds of
and immunities enjoyed by Parliament as defection, the matter will be decided by
an institution and MPs in their individual speaker or chairman or Rajya Sabha as
capacity, without which they cannot the case may be.

44
Google it:- PDF4Exams GS SCORE
https://t.me/pdf4exams «Downloaded From» https://t.me/testseries4exams

100. Correct Option: (b) are given equal representation in the


Senate irrespective of their population.
Explanation:
USA has 50 states and the Senate has 100
Rajya Sabha: members—2 from each state.
 Statement 3 is incorrect: A Constitution  Statement 4 is incorrect: The
Bench of the Supreme Court in 2006 representatives of each union territory in
upheld the Constitutional validity of an the Rajya Sabha are indirectly elected by
amendment to the Representation of members of an electroral college specially
the People Act 1951, dispensing with constituted for the purpose. Out of the
the domicile requirement for getting seven union territories, only two (Delhi
elected to the Rajya Sabha. and Puducherry) have representation
in Rajya Sabha. The populations of other
 Statement 1 is correct: The representatives
five union territories are too small to have
of states in the Rajya Sabha are elected
any representative in the Rajya Sabha.
by the elected members of state
legislative assemblies. The election is  The President nominates 12 members
held in accordance with the system of to the Rajya Sabha from people who

RE
proportional representation by means of have special knowledge or practical
the single transferable vote. experience in art, literature, science and
social service.
 Statement 2 is correct: The seats are
allotted to the states in the Rajya Sabha  The rationale behind this principle of
on the basis of population. Hence, the nomination is to provide eminent persons
number of representatives varies from a place in the Rajya Sabha without going
state to state. For example, Uttar Pradesh through the process of election. It should
has 31 members while Tripura has 1
O
member only. However, in USA, all states
be noted here that the American Senate
has no nominated members.
**********
SC
GS

GS SCORE Google it:- PDF4Exams


45

Potrebbero piacerti anche